Космос и астрономия

Ответить в тред Ответить в тред
Тред тупых вопросов №133 Leavitt edition Аноним 24/06/20 Срд 10:20:53 5852651
Henrietta-Swan-[...].jpg 18Кб, 344x450
344x450
pia15819-full.jpg 1594Кб, 3000x2400
3000x2400
expansion.jpg 396Кб, 1920x1080
1920x1080
Watch a Star Bl[...].mp4 3876Кб, 1280x720, 00:00:26
1280x720
Тред вопросов о жизни, Вселенной и всём таком.

Спрашиваем то, за что в других местах выдают путёвку в биореактор. Здесь анонимные учёные мирового уровня критически рассмотрят любые гениальные идеи и нарисованные в Paint схемы.

Предыдущий тут: >>582065 (OP)
https://2ch.hk/spc/res/582065.html

Q: Можно быстрее?
A: Можно упасть в пузырь Альбукерке, NASA уже почти надула его.

Q: Я начитался охуительных историй про уфологию, че делать, нам жопа?
A: Да, тебе жопа, можешь сгонять в зогач или куда оттуда пошлют.

Q: Что будет с человеком в вакууме без скафандра / если он упадет на черную дыру / попробует ступить на поверхность газового гиганта/солнца?
A: Он умрёт.

Q: Почему бы не привязать ракету к воздушному шару или стартовать с горы?
A: Космос - это не как высоко, а как быстро, большая часть энергии ракеты уходит на разгон вбок.
Подробнее тут https://what-if.xkcd.com/58/ (английский) https://chtoes.li/orbital-speed/ (перевод)
Аноним 24/06/20 Срд 10:28:15 5852672
>>585265 (OP)
Ебанный стыд...
Во-первых, Алькубьерре.
Во-вторых, не упасть, а создавать вокруг корабля изнутри (иначе кина не будет).
В-третьих, НАСА искривляет пространство на десятимиллионную часть, контролируя
это сверхточными интерферометрами, до самого варп-привода здесь - как до
Антарктиды раком.
Аноним 24/06/20 Срд 13:14:14 5852813
159298325425500.jpg 92Кб, 688x450
688x450
Аноним 24/06/20 Срд 15:17:14 5852914
RozalijaZalkind.jpg 67Кб, 350x440
350x440
Аноним 24/06/20 Срд 15:20:54 5852925
>>585291
Ну и нахуя оно тут?
Аноним 24/06/20 Срд 15:38:51 5852936
>>585265 (OP)
Почему Скотт Мэнли такой охуенный?
Есть ли аналогичные по информативности каналы?
Я как запускаю видео Скотта - обнаруживаю что 10 минут прошли как одна, никогда не скучно. Каждодневный астронавт как-то уныло пургу гонет и канал будто на его личности (и муска) акцентирован, а не на космических фактах.
Любопытный Дроид относительно нормальный вспомнился, и все, больше никого не упомню.
Аноним 24/06/20 Срд 15:58:24 5852997
>>585267
Если на корабле им. Аль Букерке перевезти флешку с информацией, будет ли это считаться за передачу информации быстрее скорости света?
Аноним 24/06/20 Срд 16:07:01 5853038
>>585299
Что за фиксация на флешках? Даже слово ХУЙ, написанное на борту корабля будет являться информацией. Как впрочем и отсутствие этого слова.
Аноним 24/06/20 Срд 17:34:42 5853559
>>585293
Он был бы охуенен, если бы своим ебалом меньше светил на весь экран. Я бы даже подписался, рассказывает неплохо.
Аноним 24/06/20 Срд 17:36:57 58535710
>>585355
Он не светит, когда есть чего показать.
Последнее видео про докинг адаптер - 95% хронометража - хроники со стыковочными узлами.
Аноним 24/06/20 Срд 21:56:32 58545711
Какие есть крутые двигатели, которые человечество теоретически сможет создать в этом веке? Хочется чтобы было как в сериале "Экспанисия" - летишь к любой точке в солнечной системе с постоянным ускорением в 10-15 м/с^2 на огромном корабле, который не состоит на 90% из топлива.
На ум приходит только двигатель на ании-материи, но чет я сомневаюсь, что в этом веке люди научатся ее добывать и хранить в промышленых масштабах.
Аноним 24/06/20 Срд 22:01:30 58545912
>>585457
Взрыволет на нюках.
Аноним 24/06/20 Срд 22:13:21 58546113
>>585457
Такая термоядерная хуета вполне реальна
Аноним 24/06/20 Срд 22:16:12 58546414
>>585459
Так ускорение будет импульсным. Не думаю, что будет комфортно неделю лететь в невесомости, когда тебя каждые несколько секунд колбасит об нос корабля.
Хотя, эта идея наиболее реалистична. Мб если пускать очень часто очень маленькие бомбы, то и норм будет.
Аноним 24/06/20 Срд 22:22:29 58546915
Одно из решений парадокса Ферми в том, если не ошибаюсь, что на определенном этапе развития разумные цивилизации закономерно замыкаются в себе, т.е. утрачивают интерес к видимой внешней вселенной. Что интересного почитать на эту тему?
Аноним 24/06/20 Срд 23:59:27 58551416
>>585469
Историю за последние 50 лет, вместо колонизации Луны и Марса развиваем интернеты и прочие сферы развлечений.
Аноним 25/06/20 Чтв 01:15:43 58552317
159303691802117[...].jpg 40Кб, 800x499
800x499
Аноним 25/06/20 Чтв 01:15:57 58552418
>>585469
айзек азимов конец вечности
Аноним 25/06/20 Чтв 03:40:05 58553319
>>585469
Парадокс Ферми апеллирует к идиотскому тезису, что инопланетяне с какого-то хрена будут общаться в радиодиапазоне и срать сигналами во все стороны в рамках своей программы SETI. Даже гипотетический межзвёздный лазерный модем обнаружить с Земли практически невозможно, а уж если инопланетяне используют сверхсветовой канал связи - обнаружить их нашими методами невозможно вообще никак.
Аноним 25/06/20 Чтв 04:36:29 58553620
>>585533
>Даже гипотетический межзвёздный лазерный модем обнаружить с Земли практически невозможно
В фокусе гравилинзы можно, но это надо на каждую изучаемую звезду отдельный телескоп в пояс Койпера запускать.
Аноним 25/06/20 Чтв 05:25:28 58553721
>>585514
Так вот я к тому что может как раз это и не плохо, а хорошо. Если допустить, что все пошли этим путем, так может и нам надо туда мм? Может мы просто не там ищем путь к звездам? Может надо вкладываться в ветку психофармакологии, нейрофизиологии, медитации, религии в конце концов?
Аноним 25/06/20 Чтв 05:37:16 58553822
>>585536
Они могут еще замаскировать свой канал рандомными шумами, так что без ключа ты в шумах сигнал не найдёшь, будет просто как нытье какой-то переменной звезды.
Аноним 25/06/20 Чтв 05:38:28 58553923
>>585537
Вкладывайся, дружище. Выкидывй свой компьютер и вперёд, езжай в Камбоджу на границу с Лаосом за просветлением.
Аноним 25/06/20 Чтв 06:15:45 58554024
>>585539
Ты можешь мыслить без стереотипов? Заметь, раздел называется "Космос и астрономия", а не "Наука о космосе", с этим у нас все нормально.
Я что имею в виду, собственно антропный принцип. Вселенная такая какой мы ее видим, потому что можем осознавать ее только посредством нашего разума. Следовательно, это вселенная человека до тех пор, пока мы остаемся существами с разумом человека. Тот же пример про моль, поедающую книгу - для моли реальности текста не существует, а для человека реальность текста имеет большую ценность, чем реальность бумаги. Можно ли по аналогии представить себе такую же реальность, которая будет текстом по отношению к человеческой вселенной? Вот в чем мой вопрос.
Я вписал религию в список, потому что это именно то на что пытаются ответить все религии. "Будьте как боги, и войдете в царствие небесное". Человеческий разум 2 миллиона лет развивался вообще-то, откуда это 150-летнее высокомерие "все мы изобрели радиотелескоп, все хуйня давай по новой". Не пойдет!
Аноним 25/06/20 Чтв 07:23:50 58554125
>>585540
Всё это словоблудие про психонейромидатцию лишь троянский конь для пропуска за барьеры скетицизма науки, чтобы потом из него как по плану выскакивал какой-нибудь пастух семитской религии, как ответ на все словоблудские набросы. Плавали знаем, нахуй проследуй, Ленин-гриб.
Аноним 25/06/20 Чтв 12:55:08 58555626
>>585303
Тащемта уже само наличие этого корабля в точке XYZ будет информацией.
Аноним 25/06/20 Чтв 12:59:06 58555727
>>585457
Никакие. Предпосылок к этому ровно 0, только манясхемы, которые изменятся 500 раз даже если кто-то займётся. А ванговать за пределами хоть каких-то предпосылок - все равно что рулетку крутить
Аноним 25/06/20 Чтв 13:05:26 58555928
>>585469
Ничего. Все рассуждения на эту тему - фикшен. Вымысел, основанный на куче допущений, одно из которых назвал анон >>585533 . Разве что может быть Лема, который настолько абстрактен и нематериален, что при рассуждении об этом парадоксе его даже не касается (что само по себе парадокс). Может быть Игана ещё, там хоть вымысел, но очень высокоуровневый, а не чистое фэнтези.
Аноним 25/06/20 Чтв 13:49:39 58556129
>>585537
>психофармакологии, нейрофизиологии, медитации, религии в конце концов?
И как это поможет телепортации к звездам? Может это научит видеть рентген?
Аноним 25/06/20 Чтв 13:54:57 58556230
image.png 33Кб, 350x350
350x350
>>585561
>научит видеть рентген?
Я и так его вижу, он весьма заметный в центре рентген-кабинета стоит.
Аноним 25/06/20 Чтв 16:32:53 58558531
>>585559
>Лема, который настолько абстрактен и нематериален
>>585559
>Лема, который настолько абстрактен и нематериален
Чо? Ты чо? Уж реальней Лема вообще ничего быть не может, он материалистичен до абсолюта.
Аноним 25/06/20 Чтв 18:46:42 58560532
>>585265 (OP)
ребятки, я к вам с очередной платиной пришел..
вот типа нельзя лететь далеко из-за повышенной радиации. почему нельзя создать защиту из магнитного поля, развесив по корпусу электромагниты и т.д., для создания защитного поля? чисто умозрительно, теоретически, без учета технических проблем?
Аноним 25/06/20 Чтв 18:50:14 58560633
unnamed (1).jpg 103Кб, 502x351
502x351
>>585561
>как это поможет телепортации к звездам?
Телепортация тушки и не нужна. Между телепортацией и просто межзвездным перелетом нет концептуальной разницы для данного рассуждения, гостей-то мы и так и так не наблюдаем.
О чем речь, допустим, микрожитель микросхемы изобрел телескоп. Он смотрит на окружающий мир, что он видит? Пластик, железяки, пыль, присохшую кошачью шерсть, дохлых тараканов, крепления к корпусу и т.д. - красная стрелка. Расстояния велики, достигнуть даже ближайших наблюдаемых структур в разумное время он не может. А надо разгадать свое назначение и научиться принимать и отдавать сигналы (синяя стрелка). Тогда коммуникация мгновенная, и вся система к тому же приобретает смысл.
Если что я не пытаюсь кого то в чем то убедить этой дичью, просто думаю насколько иным в принципе может быть подход. Пространственно-временной континуум тоже ведь неочевидная штука, а вон как оно вышло в итоге.
Аноним 25/06/20 Чтв 18:50:38 58560734
Ты как гамма-излучение и нейтроны примагничивать собрался?
Аноним 25/06/20 Чтв 18:51:04 58560835
Аноним 25/06/20 Чтв 18:51:06 58560936
>>585605
Во-первых, магнитное поле влияет не на любую радиацию, а только на ту, что состоит из заряженных частиц (гамма и нейтроны мимо, например). Во-вторых, подозреваю, при напряженности поля, достаточной для защиты от радиации на дистанции в пару метров, тебе просто поплохеет, пушо на тушку это тоже будет оказывать влияние.
Аноним 25/06/20 Чтв 19:16:26 58561137
>>585607
они будут ударятся о заряженные частицы в поле и остановяться
Аноним 25/06/20 Чтв 19:48:38 58561538
>>585611
Частиц там должно быть как минимум как в метровой толщине свинцовой пластине.
Аноним 25/06/20 Чтв 19:50:39 58561639
>>585615
а если они будут быстро летать то успеют зацепить все нейтроны же
Аноним 25/06/20 Чтв 21:36:44 58563040
>>585616
Как ты себе это представляешь? Нейтроны же не пешком ходят, даже тепловые нейтроны тоже летят на скорости в несколько километров в секунду. И это мы еще о гамме не начали говорить.

У тебя никак не получится сделать защиту, эквивалентную метру свинца, без запихивания в щит количества вещества, сравнимого с этим самым метром свинца.
Аноним 26/06/20 Птн 00:42:38 58565941
Аноним 26/06/20 Птн 00:52:26 58566542
>>585659
>Я так понял у кого-то новая вайфа спейсача.
Что это значит?
Аноним 26/06/20 Птн 00:57:43 58567143
Смотрите.
Берём запутанную пару. Когда мы коллапсируем одну то одновременно коллапсирует вторая. Если мы измерим вторую, то мы сколлапсируем и первую. Возьмём массив запутанных пар. Отправляем каждый день на далёкую звезду массивы с запутанными частицами. Они идут постепенно и равномерно, не обязательно на релятивистских скоростях. В один момент эта линия из массивов достигает звезды. Через определённое время, на земле решили передать информацию к звезде. Мы открываем запасники и коллапсируем пару, которая должна была прийти завтра. А в это время у далёкой звезды помнят инструкцию оставленную столетиями ранее и каждый раз при приходе посылки они измеряли пары на предмет запутанности. Каждый раз она разрушалась. Но в обозначенный день они проверили и обнаружили, что пара уже разрушена или не разрушена. Что будет обозначать информацию в 1 бит переданную на расстояния многие световые года за день.

Где ошибка?
Аноним 26/06/20 Птн 00:58:11 58567344
Аноним 26/06/20 Птн 01:13:40 58567945
>>585673
Это херня. Была другая (забыл как звали), дважды за полгода в шапке ТТВ красовалась.
Аноним 26/06/20 Птн 01:46:38 58568146
>>585671
>каждый раз при приходе посылки они измеряли пары на предмет запутанности
>Но в обозначенный день они проверили и обнаружили, что пара уже разрушена или не разрушена

Такого измерения не существует, запутанность это не какое-то отдельное свойство, а просто тот факт, что некоторые свойства двух частиц всегда коррелируют между собой. При любом измерении запутанность необратимо разрушается и обе частицы принимают противоположное состояние. Самый простой пример — спин, если у одной частицы его направление было измерено в одну сторону, то у другой всегда будет точно в противоположную.

Ты измеряешь спин частицы и он, например, направлен вверх. Это вообще ничего не говорит тебе о том, была ли до этого измерена парная ему частица, и если да, то когда. Всё, что запутанность тебе говорит, это то, что у парной частицы спин теперь направлен вниз, больше из неё нельзя извлечь ни единого бита информации. Повлиять на направление спина, не разрушив запутанность, тоже нельзя.

Короче, квантовая телепортация это просто способ получить полностью случайную, но 100% скоррелированную информацию в двух местах одновременно. Никакой полезной информации через неё передать нельзя.
Аноним 26/06/20 Птн 02:10:30 58568347
>>585681
А можно ли сделать пары пар частиц при запутанности быть похожими и при нарушении запутанности определить, что если бы не разрушили, доминировал бы какой-то спин, а при нарушенной запутанности статистически более или менее одинаковое количество и тех и других спинов?
Аноним 26/06/20 Птн 02:18:19 58568448
>>585683
Нет и нет. Запутанность только в парах частиц, а параметры полностью случайны.

Никакой информации через запутанность не передается.
Аноним 26/06/20 Птн 03:12:24 58568649
Поясните как работает краулер на КСЦ? Эта йоба что возила ололон и шаталы. Их две же всего, да?
Вот на нее ракету поставили, она неспешно поехала и заехала на стартовую площадку. Дальше че? Ракета с этой штуки и взлетает, или ее снимают и переставляют на саму площадку? Как?
И для каких ракет эти транспортеры используются, а какие на КСЦ погружают на площадку иначе?
Аноним 26/06/20 Птн 07:47:30 58569350
четырехщелевой [...].png 23Кб, 1328x766
1328x766
>>585671
Нет ошибки. Я эту схему ещё в лохматом 2011 описывал. Здесь получаются очень сложные парадоксы, от которых привычная нам реальность идёт по пизде, и появляются всякие приколы с многомерным временем.

>>585684
Пикрелейтед.
Из центрального источника влево и вправо короткими очередями выпускаются запутанные частицы. Каждая очередь это один бит. Слева экран с двумя щелями и справа экран с двумя щелями. Между экранами один световой год, но попадают частицы в них одновременно.
Когда щели свободны - очередь частиц складывается в интерференционную картину. Это "0". Когда хотя бы к одной щели хотя бы на одном экране подносится детектор - с обеих сторон очередь частиц складывается в две полосы. Это "1". Поднося и убирая детектор мы передаём информацию мгновенно.

Суть в том, что нам плевать, в какую щель прошла частица. Нам важно то, разрушена ли волновая функция или нет. Никакого запрета на "дистанционное разрушение" волновой функции нет, потому что волновая функция у запутанных частиц по определению одна единая. Нельзя узнать, что слева частица прошла через щель А, не узнав при этом, что справа частица прошла через щель А'. Интерференционная картина разрушается одновременно на двух экранах.

Здесь вот что важно. Строго говоря, это не является "механистической" передачей информации через причинность пространства-времени. Это нефизический процесс. Тем не менее, такой процесс позволяет совершить видимость полезной передачи информации с точки зрения наблюдателя, что возвращает нас к трудному вопросу, что же такое наблюдатель и что на самом деле декогерирует.
Например, с точки зрения квантового самоубийства сверхсветовая передача данных вполне возможна. Алиса пишет сообщение Бобу, измеряет запутанные частицы и стреляет себе в голову всякий раз, когда результат измерения не совпадает с желаемым битом. Таким образом, в рамках многомировой интерпретации Алиса выживает только там, где Боб получил её сообщение, хотя с точки зрения физики правильный результат у Боба получился совершенно случайно. А описанная выше схема устраняет необходимость стрельбы в голову, как бы выворачивая ящик шрёдингера наизнанку.
Аноним 26/06/20 Птн 09:52:22 58569451
1A9ktJ8.jpg 78Кб, 439x438
439x438
Почему негров в космос не запускают? Боятся сравнения с подопытными обезьянами?
Станут ли их запускать после BLM?
Аноним 26/06/20 Птн 09:52:37 58569552
>>585686
Так прямо с него и запускается, шаттлы и сатурны с MLP стартовали. Да не то блядь, а Mobile Launch Platform. Один транспортер тащит саму ракету, второй - башню обслуживания. Не используется вроде ни для чего, сейчас LC-39 только тяжелые фальконы пускают, а те горизонтально перевозят. SLS скорее всего на краулерах будет ездить уже.
Аноним 26/06/20 Птн 09:55:35 58569653
800px-GuionBluf[...].jpg 164Кб, 800x1000
800x1000
>>585694
А Гайона Стюарта Блуфорда видимо не было, да?
Аноним 26/06/20 Птн 09:57:30 58569754
800px-MaeCarolJ[...].jpg 148Кб, 800x1000
800x1000
Аноним 26/06/20 Птн 10:03:13 58569855
>>585696
Два уголька среди сотни снежков. Слишком мало.
Аноним 26/06/20 Птн 10:04:55 58569956
>>585698
Так им никто блядь не запрещает в космос летать, они сами не хотят. И эти двое еще в восьмидесятые летали, когда сегрегацию только-только выключили. Но зачем, если можно сидеть на пособиях и с 12 лет торговать крэком?
Аноним 26/06/20 Птн 11:04:18 58570157
>>585694
Когда Шаттлы летали - запускали. Летало по 7 человек за раз, по 5-10 полетов в год, могли впихнуть нескольких без особых проблем для миссии. А сейчас, когда пускают по 4 корабля в год, на которых 1, реже 2 американца, как то тяжеловато жертвовать целое кресло ради диверсити и инклюжена.
>>585698
Не 2 а 13.
Аноним 26/06/20 Птн 12:10:53 58570658
415982014Articl[...].jpg 297Кб, 1875x776
1875x776
KimDelayedChoic[...].png 67Кб, 1280x1065
1280x1065
>>585693
О, да это же почти классический delayed-choice quantum eraser.

Одна проблема — твой пикрелейтед в реальности не работает. Интерференционной картины не будет ни в том, ни в другом случае. Вернее, в одном случае будет два перекрывающихся простых горба, а в другой — две комплементарных интерференционных картины, но результат для внешнего наблюдателя будет выглядеть абсолютно идентично. Ни единого бита через такую систему передать нельзя.

>If we instead choose to herald using D1 or D2 without distinguishing between the two, there is no interference pattern. This is because we effectively ignore the polarization state of the trigger photon, leaving the signal photon in a mixed state.

https://www.nature.com/articles/srep04685
Более подробное разжевывание — https://physics.stackexchange.com/questions/179348/double-double-slit-with-entangled-photons
Аноним 26/06/20 Птн 12:53:40 58570759
>>585695
Башня аполлона не стационарная что ли была?
Как и кто перевозит тяжелые флаконы и как их ставят?
Как тебя отблагодарить за потешную шутейку про млп?
Аноним 26/06/20 Птн 12:55:57 58570860
image.png 175Кб, 510x512
510x512
>>585701
>А сейчас, когда пускают по 4 корабля в год, на которых 1, реже 2 американца, как то тяжеловато жертвовать целое кресло ради диверсити и инклюжена.
Despite making up 13% of the population...
African Americans can't achieve 13% of the astronaut crew.
Аноним 26/06/20 Птн 13:04:39 58570961
>>585707
>Башня аполлона не стационарная что ли была?
Niet, не стационарная.
>Как и кто перевозит тяжелые флаконы и как их ставят?
Транспортер с подъемником. Тащит горизонтально, подъезжает к стартовому столу, аккуратненько поднимает ракету и мягонько передает в лапки башни обслуживания.
>Как тебя отблагодарить за потешную шутейку про млп?
Я не первый про это шучу, первый раз наверное года три назад здесь же (в ТТВ) увидел.
Аноним 26/06/20 Птн 13:07:52 58571162
>>585709
>Niet, не стационарная.
А как ее не сносило нахуй при старте сатурна-то? Она ж высокая и узкая...
>Транспортер с подъемником. Тащит горизонтально, подъезжает к стартовому столу, аккуратненько поднимает ракету и мягонько передает в лапки башни обслуживания.
Кто такой, как выглядит?
Вагины-ракетовозы и дизеля на байкодроме я представляю, они примерно одинаковые для всех (обоих двух) ракет что там летают.
А с флоридского огуречника летала половина вариантов ракет мира и я не знаю как они взгромождались на постаменты...
>Я не первый про это шучу, первый раз наверное года три назад здесь же (в ТТВ) увидел.
Точно здесь?
Я тебя постоянно подъебывал ей в треде про СЛС.
Аноним 26/06/20 Птн 13:10:52 58571263
bwwnav8l28751.mp4 90Кб, 640x480, 00:00:05
640x480
В сайфае любят так три кольца изображать которые вращаются по разным осям.
Есть ли за этим какое-то физический смысл, или это просто идея "округи вращаются по разным осям" и все?
Первый раз увидел в Квейк 2, казалось впечатляюще, а сейчас когда много раз замечал - не вижу смысла.
Аноним 26/06/20 Птн 13:19:13 58571564
>>585606
анон, ты можешь свои наркоманские измышдизмы подробнее изложить?
Аноним 26/06/20 Птн 13:21:08 58571665
>>585711
>А как ее не сносило нахуй при старте сатурна-то? Она ж высокая и узкая...
Основная масса - транспортер, а он тяжелый и внизу, как неваляшка получается. К тому же, газы в основном вниз идут, а не в стороны.
>Кто такой, как выглядит?
falcon heavy transporter erector гуглишь и смотришь картинки.
>Точно здесь?
Может и не здесь, давно было.
>Я тебя постоянно подъебывал ей в треде про СЛС
Кого меня то блядь, ты здесь один сидишь. Кроме тебя только бот от макаки.
Аноним 26/06/20 Птн 13:46:22 58572366
8a23ed8637e8a86[...].gif 925Кб, 540x360
540x360
GianlucaGimini-[...].jpg 105Кб, 750x750
750x750
>>585712
Да никакого смысла нет. Это примерно как люди без художественных навыков, пытающиеся нарисовать велосипед по памяти.
http://www.gianlucagimini.it/prototypes/velocipedia.html

Поэтому немалая часть скилла художника заключается в том, чтобы думать как на самом деле устроены вещи, которые он рисует. Чтобы не получилось "ну это хуёмоё рамки гироскопа ебанём давай и сразу вжух такое астро-хуйзнаетчто, чорная дыра тип, или как там оно у ейцегаловых))0".

Да и не припомню что-то нигде, кроме Q2 да может быть Контакта, откуда этот гироскоп ебаный наверно в квейк и передрали.
Аноним 26/06/20 Птн 13:53:09 58572467
>>585605
Радиация в космосе есть от двух вещей - галактическая (прилетает снаружи гамма-излучение и частицы высоких энергий) и в поясах Земли. В радиационных поясах Земли не собственно радиация, там летают электроны и протоны на больших скоростях, они бахают в корпус аппарата и получается радиация. Вот их отклонить магнитом можно, но нахуй не нужно, пушто в поясе ты все равно не задерживаешься, его облететь можно, и пролетаешь ты быстро. А гамма-излучение - основной источник радиации при перелёте и жизни на Марсе или Луне например - ты не отклонишь магнитом. Частицы высоких энергий можно, но не нужно - они сравнительно редкие, и магнит у тебя получится ебический, жрущий прорву электричества (с генерацией которого в космосе и так проблемы)
Аноним 26/06/20 Птн 14:04:57 58572768
Apollo11SaturnV[...].jpg 3455Кб, 3590x3015
3590x3015
>>585707
>>585709
>>585711
У Сатурна две башни, одна на пусковой платформе вместе с РКН, вторая сама по себе, и обе ездили на краулерах.
У Флакона так: TEL удерживает ракету, а гидроцилиндры для подъёма встроены в пусковую установку, причём на всех СК конструкции разные. А башен для обслуживания нет.
Аноним 26/06/20 Птн 14:18:21 58572869
image.png 479Кб, 675x450
675x450
>>585716
>К тому же, газы в основном вниз идут, а не в стороны.
У них не выкопана чаша унитаза как у нас на байкодроме же.
Так что внизу идут в стороны.
>falcon heavy transporter erector гуглишь и смотришь картинки.
Гуглится только пикрил - 78 стальных колес которые везут один блок. Я буквально по твоему тексту гуглил. Мне интересно, как ее довозят и водружают наверх.
>Кого меня то блядь, ты здесь один сидишь. Кроме тебя только бот от макаки.
Ох ты дурень, я и есть бот от макаки.
Аноним 26/06/20 Птн 14:20:15 58572970
>>585723
>Да и не припомню что-то нигде, кроме Q2 да может быть Контакта, откуда этот гироскоп ебаный наверно в квейк и передрали.
Из крупных популярных медиа - считай и не было. А вот в шлаке и пародиях в 90-х предостаточно было. Это как крик Вильгельма в кино или БРРРРРАААААУУУУ в современных трейлерах.
Аноним 26/06/20 Птн 14:21:07 58573071
image.png 356Кб, 522x305
522x305
>>585727
Как ракета с краулера оказывается на этом постаменте?
Аноним 26/06/20 Птн 14:27:28 58573172
Ap10-KSC-69PC-1[...].jpg 161Кб, 965x768
965x768
Apollo11rollout.jpg 393Кб, 1500x1829
1500x1829
STS-114rollout.jpg 490Кб, 3000x1993
3000x1993
>>585730
Её прямо вместе с постаментом и везут.
Аноним 26/06/20 Птн 14:28:40 58573273
>>585731
ОХУЕТЬ.
Это ж тяжело дохуя, нет!?
И как они по наклону едут?
И как потом старт происходит, они не боятся сжечь гусеницы у транспортера?
Аноним 26/06/20 Птн 14:36:39 58573474
s67-50903cropped.jpg 1185Кб, 1387x1756
1387x1756
sJcJFyN.jpeg 518Кб, 2369x1999
2369x1999
>>585732
>И как потом старт происходит, они не боятся сжечь гусеницы у транспортера?
Краулер потом сдаёт назад и выезжает из-под платформы, разумеется. В момент запуска его там нет.
Аноним 26/06/20 Птн 14:41:21 58573575
>>585732
да, поэтому 2км они едут целый день
там особая система стабилизации, платформа наклоняется при езде в гору чтобы оставаться горизонтальной относительно земли
Аноним 26/06/20 Птн 14:42:42 58573676
>>585734
Этот момент-то поясни же. Как он сдает из-под платформы? На чем она стоит? Как этот процесс происходит?
Про это видео катастрофически мало.
Как вешают Союзы на подвесы - дохуя, я даже в прямом эфире смотрел, как ставят Протон на стол тоже. А вот как кроулер заезжает, оставляет ракету (как) и уезжает - не видел ни разу.

>>585735
Ни разу не умаляю офигенность этой темы.
Кстати, а почему они рельсы не стали юзать?
Аноним 26/06/20 Птн 14:48:44 58573777
>>585724
>не собственно радиация
В чем отличие, лол. Радиация - это просто какие-то высокоэнергетические частицы.
Аноним 26/06/20 Птн 14:48:48 58573878
Какие правила нужно держать в уме, когда пытаешься придумать обитаемую планету? Ничего страшного, что я тут спрашиваю, а не в сайфаче?
Аноним 26/06/20 Птн 14:56:23 58573979
>>585738
Зону златовласки.
Аноним 26/06/20 Птн 14:59:49 58574080
>>585736
Вот тут примерно с 11:05 показывают крепления и опоры, к которым мобильную платформу привинчивают.

https://youtu.be/3KOImkgmmk8?t=665

Просто платформа сначала поднята выше уровня опор, потом опускается ниже и встает на опоры, а потом краулер опускается еще ниже и выезжает из-под неё. Тут ничего сверхъестественного нет.
Аноним 26/06/20 Птн 15:00:27 58574181
1593172826716.jpg 48Кб, 685x567
685x567
В первую чашку наливают 50% кофе температурой 90°.

Во вторую чашку наливают 50% кофе температурой 90° и 50% молока температурой 5°.

Через 10 минут в первую чашку тоже доливают 50% молока температурой 5°.

В какой чашке будет выше температура и почему?
Аноним 26/06/20 Птн 15:05:01 58574282
>>585738
Долговременную стабильность основных параметров. Не должно быть, орбит, звезд, состава атмосферы, которые меняются за миллион лет, иначе хуй там что сложное успеет проэволюционировать.
Аноним 26/06/20 Птн 15:09:40 58574383
Аноним 26/06/20 Птн 15:10:03 58574484
>>585740
У краулера гидравлическая подвеска, спасибо.
Я балбес, сам не додумался.

Охуеть, конечно, уровень работ.

Хотя я даже не знаю, что более ебическое - монтаж платформы над кроулером со стаметровой ракетой над вами, или просто подъем стаметровой ракеты как в СССР.

Кстати - стаметровой или стометровой?
Аноним 26/06/20 Птн 15:10:34 58574585
>>585741
Недостаточно информации. Попробуйте снова.
Аноним 26/06/20 Птн 15:21:47 58574786
>>585728
>У них не выкопана чаша унитаза
Насчёт унитазов не знаю, а газоходы есть. Так как большинство РН США стартует с приподнятых мобильных платформ, то они обычно менее глубокие, чем советские, а от срезов сопел до дна расстояния не сильно отличаются.
Аноним 26/06/20 Птн 15:22:24 58574887
Почему люди ничего не делают?
Арктика почти растаяла, 60% видов вымерло за последние 40 лет, тропические леса исчезают, глобальная температура выросла why?
Аноним 26/06/20 Птн 15:29:37 58574988
Можно ли космический мусор прямо с Земли хуячить лазером? Какая мощность нужна, чтобы на НОО металл испарять?
Аноним 26/06/20 Птн 15:31:38 58575089
>>585706
>О, да это же почти классический delayed-choice quantum eraser.
Нет, это гораздо более мощный эксперимент.

В DCQE никакие запутанные частицы непосредственно в экран не летели. Летели производные от них сигнальные фотоны, сами по себе запутанностью не обладающие. Потом они складывались, а интерференционная картина восстанавливалась по производным холостым фотонам.

Здесь же всё иначе. Даун-конвертера нет. Декогерирующего деления на сигнальные и холостые фотоны нет. Есть только удвоенный двухщелевой опыт, работающий с запутанными частицами напрямую.

>Интерференционной картины не будет ни в том, ни в другом случае. Вернее, в одном случае будет два перекрывающихся простых горба, а в другой — две комплементарных интерференционных картины, но результат для внешнего наблюдателя будет выглядеть абсолютно идентично.
Нет, ты не шаришь. Отсутствие интерференционной картины в DCQE обусловлено тем, что в экран попадают уже раздекогерированные сигнальные фотоны. Они не интерферируют на друг с другом, ни сами с собой (ибо щелей перед экраном нет), они просто дают жирное пятно. Просто в этом пятне как бы зашита информация о двух взаимно-аннулирующихся интерференционных картинах, которые мы можем восстановить по регистрации холостых фотонов.

У меня на схеме ничего этого нет. С изначально рождёнными частицами вообще ничего не происходит вплоть до попадания в экран, они просто летят.
Аноним 26/06/20 Птн 15:34:46 58575190
Где сферы Дайсона?
Аноним 26/06/20 Птн 15:51:51 58575391
>>585747
>Насчёт унитазов не знаю, а газоходы есть. Так как большинство РН США стартует с приподнятых мобильных платформ, то они обычно менее глубокие, чем советские, а от срезов сопел до дна расстояния не сильно отличаются.
Газоходы есть, но в них водичка наливается, т.к. иначе отдачей ракету распидорасит. Советские унитазы не требуют налива воды, т.к. волна гасится дойдя до чаши унитаза.
Вообще сложно представить себе выкопанные чаши унитазов во флоридском побережье, их бы затопило к хуям.
Аноним 26/06/20 Птн 15:53:24 58575492
>>585748
Большинство людей - идиоты. Им похуй на мир, они думают только о себе.
У меня нет идей как с этим справляться. У тебя есть?

>>585749
Не особо. Атмосфера справляется сильно лучше, она и лазер тот самый прекрасно будет гасить.

>>585751
Никаких не обнаружено, так что ответ - в фантастике.
Аноним 26/06/20 Птн 16:16:20 58575593
>>585754
С какого расстояния мы могли бы заменить сферу Дайсона с существующими технологиями?
Аноним 26/06/20 Птн 16:16:40 58575694
07.jpg 256Кб, 1118x848
1118x848
>>585732
Не особо тяжело, пустой Сатурн 5 в сборе весит всего лишь 130т, это меньше чем средний карьерный самосвал подымает. (тащемта этот транспортер и есть переделанная платформа от карьерной техники, как у муска переделанные нефтяные платформы для посадки ступеней). У транспортера просто размеры большие, для сравнения вот эта хуйня
https://www.youtube.com/watch?v=1Wp41alE8Mg
может двигать 44 тонны на каждую ось, а собранная модулями в блок она может спокойно целый дом перевезти (в первой половине XX века вообще целые улицы двигали).

Для Н-1 тоже подобную телегу начали пилить, но чтобы ездила не по ровной насыпи, а по степям Казахстана, Зил-135Ш на моторизованных поворотных шасси от Ил-18, пикрилейтед. Вышло бы примерно как на видео, только побольше, не модульная и для бездорожья. Но потом решили делать ракету прямо на месте, и йобу бросили на стадии прототипа.
Аноним 26/06/20 Птн 16:17:33 58575795
>>585755
Заменить сферу дайсона на что? У нее из альтернатив только кольцо Нивена, наверное.
Аноним 26/06/20 Птн 16:19:19 58575896
>>585756
Спасибо за развернутый ответ.

>Но потом решили делать ракету прямо на месте, и йобу бросили на стадии прототипа.
А в чем проблема, почему изначально не предусматривалось такое решение? Почему это будто крайняя мера? Это же наоборот наиболее оптимально. Тут же собрал, протестировал и запустил.
Аноним 26/06/20 Птн 16:33:47 58576097
>>585758
Так собрать ракету это не трубопровод сварить, там высочайшая точность нужна, по части сварки стенок баков переменной толщины, монтажа движков и тд.
Аноним 26/06/20 Птн 16:36:57 58576198
>>585758
Потому что это использовать имеющийся ракетный завод в Москве vs строить новый, вместе с миграцией большого количества рабочих и спецов. Всё это в пиздец сжатые сроки.
Аноним 26/06/20 Птн 16:40:30 58576299
>>585760
Ну на Байке на самом деле была отвёрточная сборка из крупных узлов, влезавших в габарит, и сделанных на основных предприятиях, в Самаре в основном. Но да, завод неебовый всё равно получился, туда половина Прогресса переехала. Да и МИК тоже.
Аноним 26/06/20 Птн 16:46:30 585764100
>>585748
Потому что людям похуй. При этом они не идиоты, просто большинство рассуждают, что чем орать про выдуманные проблемы, луче заняться своей жизнью. Вот и живут. И, кстати, правильно, 90% всей этой алармистской хуеты (в том числе и научных статей по...) или натянуто/выдумано/подтасовано ради карьеры, либо человек идёт на поводу у циничных манипуляторов (в политике/науке) и становится идейным идиотом без критичечкого мышления и уже сам генерирует посты, статьи и разговоры.
Какой пример? "Обилие" статей о пользе глобального потепления (их нет) .
Аноним 26/06/20 Птн 16:46:41 585765101
>>585760
Это проблема почему?
Для СССР например, в чем проблема переехать станками и спецами поближе к байконуру?

>>585761
А можно строить не целиком новый а завод крупноузловой сборки который уже будет из кучи полезных напиленных деталей делать целиком модули.
У СССР разве была проблема с "миграцией большого количества рабочих и спецов"? Читал про базы на камчатке - там пиздец с нулем инфраструктуры, и довозили всяческими способами все, вплоть до ракет в последствии.
А на байконур не могли куда железка и аэродром нормально ходят?
Что-то не сходится, не понимаю.
Аноним 26/06/20 Птн 16:47:32 585766102
>>585758
>протестировал
Хех мда. Если б её тестировали перед полётом, она может быть и полетела даже. Но это было бы возможным, если б Королёв не стал городить мемный гиперболоид, а взял бы свою собственную абсолютно нормальную и рабочую вязаночную компоновку от Р-7, разбив первую ступень на блоки меньшего диаметра. Тогда может быть и доставить можно было без ёба-завода в степи, и протестировать блоки по отдельности на маленьком стенде.

Но это я с дивана из 2020 говорю, кто его знает какой там челлендж в 60-х виделся. Может с синхронизацией движков между блоками траблы были бы (они впрочем и так были, борткомпьютер заменяли на более быстрый), может с массой, может с аэродинамикой, может ещё с чем.
Аноним 26/06/20 Птн 16:48:33 585768103
>>585765
>Это проблема почему?
>Для СССР например, в чем проблема переехать станками и спецами поближе к байконуру?
Дорого. Ракетный завод же не просто отдельно стоит, он зависит еще от кучи предприятий. Да и под спецов надо отдельный город строить со всей инфраструктурой. И все это ради одной Н-1, которая слетала бы пару десятков раз.
Аноним 26/06/20 Птн 16:48:59 585769104
>>585765
Ну говорить легко, а сроки жмут и мешки ворочать надо. Ну довозили, и собственно так и сделали с Н-1 - завод отвёрточной сборки.
Аноним 26/06/20 Птн 16:54:46 585771105
>>585766
>взял бы свою собственную абсолютно нормальную и рабочую вязаночную компоновку от Р-7, разбив первую ступень на блоки меньшего диаметра. Тогда может быть и доставить можно было без ёба-завода в степи, и протестировать блоки по отдельности на маленьком стенде.
Подробней, пожалуйста.

>>585768
>>Для СССР например, в чем проблема переехать станками и спецами поближе к байконуру?
>Дорого.
Когда это было проблемой для СССР? Глянь на хранилища боеголовок, на подводные верфи, на массивы антенн в тундре.
Аноним 26/06/20 Птн 16:58:04 585772106
>>585771
>Когда это было проблемой для СССР? Глянь на хранилища боеголовок, на подводные верфи, на массивы антенн в тундре.
Все тобой перечисленное - это надолго. А проворачивать такую многоходовочку из-за одной ракеты - просто бессмысленно.
Аноним 26/06/20 Птн 16:58:58 585773107
image.png 193Кб, 1123x784
1123x784
>>585772
Иронично, не находишь?
Аноним 26/06/20 Птн 17:01:59 585774108
>>585771
>Подробней, пожалуйста.
А чего тут подробней? Просто увеличенную Р-7 бы запилил, и боковушки тестировал отдельно на меньшем стенде. Н-1 потому и не полетела никуда, потому что её не испытывали перед полётом в сборе. Ну и потому что Мишин оказался долбоёбом. Для такой широченной ступени нужен был ебовый стенд, на него денег уже не хватало, поэтому испытывали по старинке - в полёте. (ну кроме прожига движков по отдельности). У сатурна же наоборот, вложились в испытания по полной, в результате он полетел, а Н-1 нет. Позже в Энергии это всё учли, и она полетела сразу, хоть и не без проблем с оплавленным стартовым комплексом.
>Когда это было проблемой для СССР?
Всегда. Ресурсы только выглядят бесконечными, но такими не являются. Тащемта на Н-1 даже близко столько бабла не дали, сколько Королёв просил, и то не в срок. Всё было именно что перекинуто на срочное клепание МБР, потому что штаты тоже охуели со Sputnik'а, срочно подорвались и наклепали себе несколько сотен Титанов.
Аноним 26/06/20 Птн 17:06:32 585776109
>>585771
>Когда это было проблемой для СССР? Глянь на хранилища боеголовок, на подводные верфи, на массивы антенн в тундре.
Ты сравниваешь военку с гражданским космосом. Лунный полёт лишь в штатах был общенациональной программой, на которую мобилизовали вообще всю промышленность страны. В совке он никогда таким не был, лишь в 64 году бабла дали побольше чем обычно, но общенациональной программы не случилось.
Аноним 26/06/20 Птн 17:09:14 585777110
>>585776
>рограммой, на которую мобилизовали вообще всю промышленность страны. В совке он никогда таким не был, лишь в 64 году бабла дали побольше чем обычно, но общенациональной программы не случилось.
Тогда странно как преуспели вообще по всем параметрам кроме высадки человеков на луну.
Аноним 26/06/20 Птн 17:21:11 585778111
>>585777
Ну потому что муриканцы неторопливо дрочили бибу, планируя где-нибудь лет через 30 полетать, пока ОКБ-1 в темпе клепало свою Р-7, которая к тому же оказалась довольно удачной для РКН (и неудачной для МБР, своей главной задачи). Запуск Жажарина был реализован сравнительно небольшими силами по дальнейшим меркам, просто это было новаторство. А потом они прослоупочили модульные станции, потому что все ресурсы пустили на провальный Шаттл и наяривали на оказавшуюся невыгодной концепцию ручного управления, в результате просрали совкам создание космических роботов и модульных станций, да и вообще буксовали на месте 20-30 лет в пилотируемой космонавтике. Человек, роботы, строительство на орбите, долговременное проживание в космосе - это то, в чем совки преуспели. В АМС штаты смогли значительно лучше, тогда как совки смогли лишь в Венеру, ибо аппараты тупо ломались даже на полугоде перелета до Марса, не говоря уже об аналогах Пионеров. По прикладным тоже сравнения никакого - TDRS, кейхолы и прочий ДЗЗ, ебовый сигинт и т.п., а всё по той же причине, неспособность в длительный срок существования беспилотных аппаратов.
Аноним 26/06/20 Птн 17:24:33 585779112
>>585778
Я про ключевые моменты космической гонки.
Провал в луне был сдачей в этой гонке.
Аноним 26/06/20 Птн 17:27:04 585780113
>>585753
>в них водичка наливается, т.к. иначе отдачей ракету распидорасит
Также для защиты от акустической нагрузки и для уменьшения температуры.
>унитазы
У тебя необычная фиксация. Не, правда, раньше такого тут не встречал.
>не требуют налива воды
В пусковой установке семёрок система газовой защиты - эжекция воздуха. В ПУ Энергии водяная завеса. В УКСС, ПУ Зенита-2 и Ангары подача воды прямо в выхлопные струи. В ПУ Протона брезентовые защитные экраны.
>сложно представить себе выкопанные чаши унитазов во флоридском побережье, их бы затопило
Поэтому там стартовые сооружения не заглублённые, а приподнятые.
Аноним 26/06/20 Птн 17:29:04 585781114
>>585773
Союзы не строят на космодроме, а только соединяют блоки ступеней.
Аноним 26/06/20 Птн 17:29:11 585782115
>>585780
>Также для защиты от акустической нагрузки и для уменьшения температуры.
Ты ничего нового не написал.
Аноним 26/06/20 Птн 17:30:13 585783116
>>585781
Тупарь ебаный, ты выбираешь только удобные себе хуйни и споришь с ними, чмо сраное?
Сука, я прав. Смотри еще раз. СМОТРИ.
ДАВАЙ.
Еблан
Аноним 26/06/20 Птн 17:30:33 585784117
>>585773
Так ступени ракет семейства Р-7 приезжали готовыми, их только соединяли, тому що они в габарит вписывались. А крупные ракеты пришлось бы с нуля делать на месте.
Аноним 26/06/20 Птн 17:32:17 585785118
>>585779
Ну и я про ключевые моменты. Человек, стыковки, Луна, АМС, станции - вполне были ими. Только шаттлобураны не были частью гонки, пожалуй: шаттл делали не ради рекордов, а в основном ради дешевизны и объемов (а потом он оброс требованиями и получилось то что получилось). А буран делали чтобы воспроизвести возможности, и похуй что там у них получилось.

Хотя прямой гонкой это вообще сложно назвать, каждая сторона намеренно пыталась делать то, что гарантированно не могло в данный момент получиться у другой. Совки например вообще изначально хотели Венеру огибать космонавтами, вместо того чтобы на Луну лететь. Потому что это виделось выгодней в плане обучения методичному изучению солнечной системы, т.к. надо было собирать корабль на орбите, а лунная миссия была в этом плане неинтересная, простой флаговтык. Аналогично со станциями, аналогично с человеком - противоположная сторона либо не хотела, либо была отвлечена и не могла прямо сейчас сделать подобное, либо всё вместе.
Аноним 26/06/20 Птн 17:35:16 585786119
>>585728
>Гуглится только пикрил - 78 стальных колес которые везут один блок.
О, это ж тот транспортер с видоса >>585756
Аноним 26/06/20 Птн 18:01:12 585788120
>>585755
Сферу Дайсона обнаружить невозможно. (Разве что если гравитационный ультрателескоп в фокусе солнечной линзы делать и в хайрезе конкретный участок неба рассматривать.)

Из-за сферы Дайсона звезда либо не видна совсем, либо в той или иной степени видна только с полюсов, при этом наблюдаемая яркость звезды не меняется.
Аноним 26/06/20 Птн 18:05:47 585789121
>>585788
Хер там, возможно. В инфракрасном будет фонить сильно.
Аноним 26/06/20 Птн 18:10:57 585790122
>>585789
И будет ничем не отличаться от остывающего чёрного карлика или небольшой звезды, скрытой туманностью.
Аноним 26/06/20 Птн 18:13:24 585791123
>>585790
У остывающего черного карлика светимость очень маленькая будет. А туманность, в которой звезда, сама отражает и рассеивает ее свет, так что это сразу заметно.
Аноним 26/06/20 Птн 18:21:55 585796124
>>585757
А как насчет полностью разобрать звезду, и жечь её уже самостоятельно в своем реакторе/добыть энергию из её составляющих другими способами?
Аноним 26/06/20 Птн 18:25:01 585797125
>>585790
Естественных черных карликов еще не существует, если что, их в нашу Вселенную только через пару сотен миллиардов лет завезут.
Аноним 26/06/20 Птн 19:05:06 585802126
>>585796
>разобрать звезду
Это хотя бы в теории возможно?
Аноним 26/06/20 Птн 19:07:08 585804127
>>585802
В теории - хули б нет.
Аноним 26/06/20 Птн 19:10:40 585807128
>>585804
И как? Методы какие?
Аноним 26/06/20 Птн 19:14:36 585808129
>>585807
Щас бы теория тебе предлагала практические методы.
Аноним 26/06/20 Птн 19:17:36 585811130
>>585808
Я именно практические методы спрашиваю.
У нас манямирок где все работают в юнификации, похуй на загрязнение, будем хуячить совместно как можем.
Вот вся планета объединена и что она может сделать с солнцем?
Аноним 26/06/20 Птн 19:20:01 585812131
>>585811
>Я именно практические методы спрашиваю.
Ну, тогда не спрашивай: "В теории возможно?".
Аноним 26/06/20 Птн 19:25:45 585813132
image.png 26Кб, 1140x326
1140x326
>>585812
Почему?
Я именно это и спрашиваю, красная часть на этой картинке.
Аноним 26/06/20 Птн 21:25:22 585836133
Из-за радиации Юпитера, будет ли УКВ-радио работать на поверхности Ио на дистанции 1км, 5км, 10км? Или там будет всё помехами забито? Какая связь там возможна вообще?
Аноним 26/06/20 Птн 21:26:40 585837134
Если я подпрыгну на Луне, то на какой высоте прыжка вращение Луны подо мной заставит меня приземлиться в стороне от места откуда я подпрыгнул?
Аноним 26/06/20 Птн 21:44:05 585846135
Почему мы еще не полетели на Марс?
И что там из интересного есть вообще?
Аноним 26/06/20 Птн 22:09:54 585849136
>>585846
>Почему мы еще не полетели на Марс?
Дораха.
>И что там из интересного есть вообще?
Хуй знает. А лететь, чтобы узнать (роботы могут оче мало) дораха.
Аноним 26/06/20 Птн 22:23:44 585850137
>>585849
>роботы могут оче мало
Мясной мешок, плес.
Аноним 26/06/20 Птн 22:30:13 585851138
>>585850
Как там Инсайт, закопался уже хотя бы на метр? Человек бы за пять минут его в землю ногой забил.

За сколько робот смог бы починить крыло лунного ровера с помощью скотча и такой-то матери? За месяц управился бы?
Аноним 26/06/20 Птн 22:51:17 585853139
>>585851
лол а в реальности космонавты по 2 часа тратят чисто чтобы добраться от шлюза до соседнего отсека
Аноним 26/06/20 Птн 22:56:56 585855140
>>585851
Как вы заебали тыкать своим инсайтом и буром. Запомните блять, InSight это миссия класса дискавери, т.е. литералли бюджетная (по меркам JPL, естественно) хуйня для ровно одной узкой задачи, из говна, палок и изоленты. Он в 5 раз меньше куриосити должен был стоить на самом деле в 2.5 получилось, как обычно. Он тупо не заточен под универсальность. И ещё представь, сколько тем же людям стоило бы запустить туда человека с этой ногой.

>За сколько робот смог бы починить крыло лунного ровера с помощью скотча и такой-то матери?
Если бы у него был автономный манипулятор - быстро.

Так-то современные UGV на земле вполне себе автономно ездят по пересечёнке, в роверы такая автономность пока ещё не попала, из-за медленного цикла разработки. И я абсолютно легко представляю себе робота, колесящего хотя бы на 20км/ч по поверхности Марса и мапящего всё вокруг. Ясен хуй человека это полностью не заменит, но скажем заSLAMить лидаром место будущей посадки сверху донизу в квадрате 10х10км такая штука может гораздо лучше человека, чтобы не садиться вслепую. Либо летающий дрон.

>>585846
Потому что
- техническая неготовность, надо отрабатывать дохуя чего, прежде чем лететь
- та система, которая может это сделать, слишком размылась за десятилетия, просит слишком дохуя денег, и работает без гарантии результата, поэтому на мегапроекты ей никто не даёт
- та система, которая в теории может сделать быстрее и дешевле, ещё толком нихуя не сделала, и неизвестно, не ожиреет ли пока будет делать
>И что там из интересного есть вообще?
Для быдла - жизнь ищут. А реально - просто удовлетворяют собственное любопытство за чужие деньги. (aka "потому что это охуенно")
Аноним 26/06/20 Птн 23:02:01 585856141
>>585853
ВКД на МКС это как раз тот случай, где космоняки теряют всякие преимущества перед манипуляторами. Потому что обезьяньи пальцы в перчатках уже не такие ловкие, а манипулятором можно и с Земли и с самой станции управлять с минимальной задержкой, да и набор инструментов получше пальцев можно запихнуть туда. (На МКСовских манипуляторах такого нет, но необходимость в ВКД всё равно сильно сократилась, когда манипуляторы нормально отработали)
Аноним 26/06/20 Птн 23:03:42 585858142
Сколько нужно времени для создания 1 кг. антивещества с нашими технологиями? Ценой пренебречь.
Аноним 26/06/20 Птн 23:07:15 585860143
>>585858
Нисколько, сразу килограмм антивещества невозможно создать/удержать в принципе.
Аноним 26/06/20 Птн 23:08:25 585861144
Аноним 26/06/20 Птн 23:11:15 585862145
>>585837
На любой, даже 1мм. Ведь ты не указал насколько ты хочешь переместиться.
Так-то поверхность Луны движется чуть больше чем 4.5м/с.
Аноним 26/06/20 Птн 23:12:02 585863146
>>585861
Да, полностью автономный робот и телеуправляемый робот - сильно разные вещи
Аноним 26/06/20 Птн 23:18:33 585865147
Аноним 26/06/20 Птн 23:21:32 585866148
>>585860
Я не писал сразу килограмм.
Аноним 26/06/20 Птн 23:24:00 585867149
1518959399123.jpg 85Кб, 553x674
553x674
>>585866
Лол, а как ты тогда собрался считать этот килограмм? Несколько атомов на несколько миллисекунд, затем ещё, ещё и ещё, и таким макаром в сумме насчитать килограмм?
Аноним 27/06/20 Суб 00:22:20 585876150
>>585865
Ну по крайней мере науке не известны такие процессы, по которым они могли бы образоваться и остыть за время жизни Вселенной. Скорее всего, их и правда нет.
Аноним 27/06/20 Суб 00:28:57 585879151
Аноним 27/06/20 Суб 00:52:55 585880152
>>585876
> за время жизни Вселенной.
Есть теории эволюции вселенной, в которых она значительно старше.
Аноним 27/06/20 Суб 00:58:14 585881153
>>585880
Это не теории, а гипотезы, которые ничем не подтверждены, в отличие от теории Большого Взрыва.
Аноним 27/06/20 Суб 01:15:16 585883154
>>585853
> а в реальности космонавты по 2 часа тратят чисто чтобы добраться от шлюза до соседнего отсека
Этот пост заставил задуматься о вот чем: бывало же такое, что человек вышел в открытый космос через один шлюз и зашел в корабль/станцию через другой?
Интересует в первую очередь разные входы на космической станции, что вышел тут, зашел в другом месте, но и случаи когда вышел из мира и зашел в шатал тоже интересны.
Какая инфа про это есть?
Аноним 27/06/20 Суб 01:16:41 585884155
>>585883
Ну как минимум лунные миссии, если не ошибаюсь. Или ты сейчас скажешь, что их не было?
Аноним 27/06/20 Суб 01:23:47 585885156
>>585855
>Как вы заебали тыкать своим инсайтом и буром.

Просто уж очень смешной обсёр получился, причем даже не сам изначальный фейл, а то, как этот бур целый год пытались поглубже в дырку запихнуть, а он все время обратно выскакивал.
Аноним 27/06/20 Суб 01:42:56 585886157
>>585855
>И я абсолютно легко представляю себе робота, колесящего хотя бы на 20км/ч по поверхности Марса и мапящего всё вокруг. Ясен хуй человека это полностью не заменит, но скажем заSLAMить лидаром место будущей посадки сверху донизу в квадрате 10х10км такая штука может гораздо лучше человека, чтобы не садиться вслепую. Либо летающий дрон.
Робот будет стоить миллионы долларов просто из-за цены доставки, понимаешь?
Даже джумба на марсе будет стоить миллионы долларов.
А когда у тебя такая цена за килограмм - имеет смысл запихать в этот килограмм максимум пользы.
У тебя там реально может джумба кататься и сметать щебенку пока не ебанется вниз в ущелье валес маринерис.
А ты можешь чуть доплатить и эти же миллионы долларов потратить на то, чтобы запустить туда масс-спектрометр, лазер, обсерваторию, сейсмодатчики и прочий набор инструментов.
И ты, сука, захочешь побольше инструментов запихать, т.к. полет на марс это тебе не полет в париж, это, сука, историческое событие до сих пор.
И ты после этого спрашиваешь.
Ха.

>- техническая неготовность, надо отрабатывать дохуя чего, прежде чем лететь
Инди ракетчики? Да. Роскосмос и НАСА? Опыт есть. Их не сложность останавливает.
>- та система, которая может это сделать, слишком размылась за десятилетия, просит слишком дохуя денег, и работает без гарантии результата, поэтому на мегапроекты ей никто не даёт
Про деньги ты помнишь.
На деле научный выхлоп от спама спиритами был бы охуенный.
>- та система, которая в теории может сделать быстрее и дешевле, ещё толком нихуя не сделала, и неизвестно, не ожиреет ли пока будет делать
Она есть хотя бы на бумаге?
>>И что там из интересного есть вообще?
>Для быдла - жизнь ищут. А реально - просто удовлетворяют собственное любопытство за чужие деньги. (aka "потому что это охуенно")
Жизнь ищут из фундаментальных принципов.
Ареология еще позволяет иметь новый взгляд на геологию. Напрямую "для быдла" непонятно, но затухающая ареосейсмология может помочь более точно предсказывать геологические события.
Аноним 27/06/20 Суб 01:45:22 585887158
Аноним 27/06/20 Суб 01:59:19 585888159
>>585884
>Или ты сейчас скажешь, что их не было?
Блядь, мы же нормально общались, нахер ты меня так оскорбляешь с нуля?
Конечно были.
Но переход астронавтов у них был через каплинг, через шлюзовой отсек, разве нет?

Я в ахуе на самом деле. Я запросто переношу "ты хуесос, мамку ибал" и прочие. Но считать меня противником лунной программы - это ты по больному ударил.
За что, скажи?
Аноним 27/06/20 Суб 02:18:45 585889160
>>585888
Потому, что здесь утверждать, что пиндосы не летали, и нести всякую хуйню про эфир это скорее норма, чем исключение.
Аноним 27/06/20 Суб 02:24:19 585890161
>>585889
Да вы охуели в конец.
Репортьте петухов которые такое несут. Каждый репортьте.
Кефиробляди вроде как вполне вне доски.
Пораша да, пиздецкая, надо с ней чего-то делать.
Аноним 27/06/20 Суб 02:25:50 585891162
>>585890
Кефирщиков банят, да. А поккокмоков, которые визжат про высадку на луне я репорчу, и меня банят, лол.
Аноним 27/06/20 Суб 02:26:01 585892163
Аноним 27/06/20 Суб 02:29:51 585893164
>>585891
Надо что-то сделать со спасех тредом (и его антагонистом, само собой).

>>585892
Погоди, "вконец" слитно что ли надо писать?
Аноним 27/06/20 Суб 02:34:21 585894165
Аноним 27/06/20 Суб 02:37:12 585895166
>>585888
>Но переход астронавтов у них был через каплинг, через шлюзовой отсек, разве нет?
Наверно, я скорее всего с советской нелетавшей миссией перепутал.
Аноним 27/06/20 Суб 02:39:31 585896167
>>585894
Точно. Примеры использования помогли.
Спасибо.
>>585895
>Наверно
Наверное?
>я скорее всего с советской нелетавшей миссией перепутал.
Это с которой?
Аноним 27/06/20 Суб 03:27:41 585898168
>>585883
>бывало же такое, что человек вышел в открытый космос через один шлюз и зашел в корабль/станцию через другой?
"14 января 1969 года с Байконура стартовал "Союз-4" с Владимиром Шаталовым, а на следующий день - "Союз-5", на борту которого находились командир Борис Волынов, бортинженер Алексей Елисеев и инженер-исследователь Евгений Хрунов. Тогда корабли не имели переходного люка в стыковочном агрегате. Поэтому с борта на борт - через открытый космос."
Аноним 27/06/20 Суб 03:32:35 585899169
>>585886
>Инди ракетчики? Да. Роскосмос и НАСА? Опыт есть. Их не сложность останавливает.
Опыт чего? Я о том, что в куче аспектов полета к Марсу (автономный полет на много месяцев, в минимальном варианте) даже конь не валялся, принципиально, и никто не чешется. Причем тут ракетчики? Это наименьшая из проблем.
Аноним 27/06/20 Суб 03:33:23 585900170
>>585889
Че несет, вообще охуеть
Аноним 27/06/20 Суб 03:38:05 585901171
>>585778
>аппараты тупо ломались даже на полугоде перелета до Марса
Ага, а на Венеру они телепортировались мгновенно. Но так то ты прав, это всё следствия цап-царапинга в электронике, из-за чего собственные разработки начали вылезать из говна только в середине 80-х. И затоптаны ещё глубже в говно в начале 90-х, где и находятся по нынешний день.
Аноним 27/06/20 Суб 08:17:45 585914172
>>585901
На Венеру лететь в 2 раза быстрей.
Аноним 27/06/20 Суб 13:36:15 585923173
>>585898
Во, офигенно, то что хотел узнать. А сейчас, в современности такое проводят?
Кстати, при современных ЕВА все с пуповиной вылазят же да?
Аноним 27/06/20 Суб 13:37:19 585924174
>>585899
Опыт полета к красной планете.
Туда лететь дорого и долго.
Каждая попытка на счету.
Аноним 27/06/20 Суб 14:59:18 585935175
>>585923
>сейчас такое проводят?
Нет. Нет необходимости.
>при современных ЕВА все с пуповиной вылазят же да?
Только с перецепными фалами.
Аноним 27/06/20 Суб 15:01:07 585936176
>>585935
>Только с перецепными фалами.
Это совершенно разумно и логично.
У амеров есть SAFER при ЕВА в любом случае.
А у русских?
Аноним 27/06/20 Суб 15:33:10 585948177
>>585896
>Это с которой?
Пилотируемым полётом на Луну на Л3. Сначала переход из ЛОК в ЛК через открытый космос, притом и через люк в переходном отсеке, соединяющим ЛОК с блоком Д и внутри которого ЛК; потом из ЛК в ЛОК после их стыковки.
>>585936
>А у русских?
Так же, разумеется.
Аноним 27/06/20 Суб 15:37:29 585950178
image.png 7363Кб, 1944x2592
1944x2592
>>585948
>Пилотируемым полётом на Луну на Л3. Сначала переход из ЛОК в ЛК через открытый космос, притом и через люк в переходном отсеке, соединяющим ЛОК с блоком Д и внутри которого ЛК; потом из ЛК в ЛОК после их стыковки.
Нихуя себе.
>>А у русских?
>Так же, разумеется.
Почему так же? Скафандры и системы свои же.
Американцы на орланы вешают САФЕР что ли?
Аноним 27/06/20 Суб 15:56:53 585953179
Меня серьезно беспокоит один вопрос.
А именно, что человечество будет делать, когда уран кончится?
Не секрет, что АЭС - это просто, быстро, надежно, безопасно, удобно и экологично.
Люди будут все больше использовать уран, но он не бесконечен.

Однажды людям потребуется создавать автономные колонии в очень далеких ебенях вроде Плутона, куда энергия от солнца уже не доходит.
Или вообще при межзвездных миссиях.
И вот там-то как раз и нужно питание от радиоизотопов.

Но к тому времени тупые людишки израсходуют весь уран и будут сосать бибу в космосе.
Что тогда делать?

Разве что строить фантасмагорические заводы на Меркурии по искусственному синтезу урана из более легких элементов.
Как эта проблема будет решаться в теории?

Аноним 27/06/20 Суб 16:00:54 585955180
>>585950
>САФЕР
Аналог для Орлана вроде как есть, но Роскосмос им не особо заинтересован, так что пока не используется.
Аноним 27/06/20 Суб 16:02:49 585956181
>>585953
>А именно, что человечество будет делать, когда уран кончится?
>Не секрет, что АЭС - это просто, быстро, надежно, безопасно, удобно и экологично.
>Люди будут все больше использовать уран, но он не бесконечен.
1 - Торий.
2 - переработка ОЯТ.
95% оят пригодна для генерации тепла в реакторах.
Ее не перерабатывают т.к. дешевле намайнить свежий уран.

>Однажды людям потребуется создавать автономные колонии в очень далеких ебенях вроде Плутона, куда энергия от солнца уже не доходит.
>Или вообще при межзвездных миссиях.
>И вот там-то как раз и нужно питание от радиоизотопов.
Я очень рад твоему оптимизму. Но я его не разделяю. Современные тенденции показывают, что люди даже на марсе не высадятся в нашу жизнь.

>Но к тому времени тупые людишки израсходуют весь уран и будут сосать бибу в космосе.
>Что тогда делать?
Не израсходуют. Нефть легкодоступную за сто лет израсходуют. Ее цена взлетит. Уран пока что весьма доступен.
>Разве что строить фантасмагорические заводы на Меркурии по искусственному синтезу урана из более легких элементов.
>Как эта проблема будет решаться в теории?
Никак, у тебя очень превратное мнение.
Аноним 27/06/20 Суб 16:03:31 585958182
>>585955
Космонавты выходят на одной пуповине и фалах не имея реактивной страховки?
Аноним 27/06/20 Суб 16:14:50 585962183
>>585958
Нет никакой пуповины, всё нужное в самом скафандре.
Аноним 27/06/20 Суб 16:16:01 585963184
>>585962
Когда перестали пуповину использовать?
Аноним 27/06/20 Суб 16:19:09 585964185
image.png 1897Кб, 900x1200
900x1200
Насколько ярко светятся аккреционные диски чёрных дыр (если это имеет значение — примерно солнечной массы)?
Можно ли их визуально спутать с обычными звёздами?
Аноним 27/06/20 Суб 16:23:38 585967186
>>585964
>Можно ли их визуально спутать с обычными звёздами?
Это точка которая ярко светится. Можно, конечно.
Аноним 27/06/20 Суб 16:27:06 585968187
>>585967
Ну там, допустим, зная, что расстояние до ЧД условных 2000 световых лет и масса сопоставимая с солнечной, мы будем видеть её как ещё одну звезду, или она будет неадекватно тусклой (или наоборот яркой) и выбьется из общего ряда?
Аноним 27/06/20 Суб 16:31:09 585969188
>>585968
Она будет еще одной звездой пока ты не будешь использовать спектроскопию и продвинутые методы исследования.
Аноним 27/06/20 Суб 17:00:03 585974189
Итак - ищу компанию, для пошива скафандра для выдры
Аноним 27/06/20 Суб 17:17:24 585975190
>>585974
Докажи серьезность намерений.
Аноним 27/06/20 Суб 21:53:34 586001191
Отменяли ли запуск шаттла после зажигания ССМЕ?
Аноним 27/06/20 Суб 21:58:30 586003192
image.png 1937Кб, 1886x1686
1886x1686
Если бы финансово и политически ничего не мешало, могла ли быть йоба-Энергия?
Аноним 27/06/20 Суб 22:10:05 586006193
>>586003
То, что было полезно от Энергии стало Зенитом и Атласом-5, а остальное (РД-0120 и всякое с водородом) нужно лишь для сверхтяжа, который после этого был не особо нужен. Единственное что можно было на Энергии-М выводить Мир-2, но вышло как вышло.
Аноним 27/06/20 Суб 22:30:14 586008194
Аноним 27/06/20 Суб 23:18:45 586012195
>>585867
А только на несколько миллисекунд?
Аноним 28/06/20 Вск 01:50:50 586023196
>>586008
Охуеть.

А вот эти искры, это для поджига водорода да?
Чем искры генерируются, как такая фигня работает? Там же не сидят огромные кремневые крутяшки из зиппо?
Аноним 28/06/20 Вск 03:38:20 586028197
>>585975
Я назвал выдру - Гагарин
Аноним 28/06/20 Вск 03:43:10 586029198
5ff6f84s-960.jpg 109Кб, 537x800
537x800
>>586003
>Если бы финансово
Ну ты понял. А реально - нет задач при такой стоимости, слишком дорогая одноразовая хрень. Модульность мнимая, как и у Ангары - при изменении нагрузки должно меняться и соотношение масс ступеней, а набирать вязанки с мелкими баками плохо по куб-квадрату. Настоящие модульность и дешевизна могли бы быть у Н-1, вот эту ветку зря забросили. А если бы туда еще и посадку ступеней, эх..
Аноним 28/06/20 Вск 03:53:53 586030199
i.jpg 29Кб, 465x320
465x320
6041big.png 1062Кб, 723x768
723x768
002all-topf6e8q[...].jpg 80Кб, 900x500
900x500
>>586023
В Хогвартсе заказывают.
Аноним 28/06/20 Вск 08:01:07 586032200
>>586003
Экспедиция на Марс.
Аноним 28/06/20 Вск 12:04:40 586042201
15754718606540.jpg 82Кб, 1200x1200
1200x1200
Является ли толстая жирная атмосфера необходимым условием для жизни на планете?
Если бы не толстая земная атмосфера, не было бы защиты от солнечной радиации, а чем это чревато для поверхности и так ясно.Так же при атмосфере, близкой к нулю жидкости испаряются (и не речь не только о воде)
Значит ли это, что искать жизнь в пределах нашей системы можно только на Земле (логично), каких-нибудь экстремофилов или неуглеродные организмы на Венере, а так же на Титане, большой луне Сатурна? И, возможно, немножнчко Марс,но там вообще тухло в этом плане
Аноним 28/06/20 Вск 13:20:10 586047202
время оборота планеты, за которое сменяются день и ночь,каким образом влияет на планету?
возмём несколько земель. одна - наша, обычная стандартная. другая вращается вокруг своей оси за какие-нибудь часа 4. третья земля находится в приливном захвате с газовым гигантом, которому приходится луной и, соответственно, один суточный цикл,то есть смена дня и ночь, будет соответствовать месяцу, то есть обороту вокруг своего юпитера, что по времени начинается от пары дней
а на что это влияет? и как?
Аноним 28/06/20 Вск 14:13:43 586052203
>>586042
Я думаю, что моя выдра - выживет где угодно.
Аноним 28/06/20 Вск 14:21:18 586055204
>>586047
При прочих равных при долгом дне суровее климат. За день поверхность нагревается, за ночь - остывает. Чем они дольше, тем больше перепады.
Аноним 28/06/20 Вск 15:09:45 586060205
>>586042
Думаю, что какая-то жижка все же нужна.
Не обязательно вода, но что-то должно быть жидким.
Аноним 28/06/20 Вск 15:57:37 586066206
Аноним 28/06/20 Вск 16:00:48 586067207
Аноним 28/06/20 Вск 17:27:55 586074208
>>586055
>При прочих равных при долгом дне суровее климат
А вот и наоборот, хоть это и неочевидно. Потоки влажного воздуха стабилизируются, на ночной стороне вечный дождь и оттуда текут реки и моря на дневную, чтобы испариться под солнцем, температура по всей планете выравнивается как в тепловой трубке.
Аноним 28/06/20 Вск 17:43:04 586077209
>>586074
Это, может, если в приливном захвате со звездой. Если смена суток есть, но длительная, то хуй.
Аноним 29/06/20 Пнд 01:31:41 586122210
А солнце щас жжёт углерод или что-то тяжелее водорода, хоть в каких-то количествах, или нет? И оно щас горячее просто потому, что из-за большей концентрации гелия жечь водород труднее и приходится сильнее греться?
Аноним 29/06/20 Пнд 01:43:58 586123211
>>586122
Там, по-моему, такие "зазоры" между условиями для реакций, что ничего кроме водорода оно жечь не может.
>И оно щас горячее просто потому, что из-за большей концентрации гелия жечь водород труднее и приходится сильнее греться?
Ты имеешь в виду, по сравнению со старыми добрыми временами? Там, вроде, принцип немного другой. Гелия со временем становится больше, поэтому ядро уплотняется и водород сжигается эффективнее и быстрее. Но я не настоящий сварщик, мог и напиздеть.

Алсо, температура внешней оболочки (около 6000 К у Солнца) напрямую не связана с температурой ядра, где, собственно, все реакции и идут. Например, когда Солнце станет красным гигантом, температура оболочки упадет куда-то в район 3000-4000 К, зато в ядре вырастет в разы (а то и на порядки может быть, надо гуглить), чтобы жечь гелий и далее по списку.
Аноним 29/06/20 Пнд 01:59:35 586124212
>>586122
>>586123
Не знаю, насколько это можно считать жжением углерода, но CNO-цикл на Солнце работает, пусть и вклад его в общее энерговыделение что-то около одного процента. Но углерод-азот-кислород там не расходуются, суммарная реакция все равно та же самая — синтез гелия из водорода.

В ОП-посте треда №129 неплохое видео было на эту тему.
Аноним 29/06/20 Пнд 09:19:11 586127213
>>585802
Алё, звёзды сами себя разбирают непрерывно, нужно только этот процесс чуть-чуть ускорить.
Аноним 29/06/20 Пнд 09:58:41 586128214
>>585811
>Я именно практические методы спрашиваю.
Сфера/рой Дайсона с отражающей внутренней поверхностью, например. Звезда перегревается и хлопает, и собирай себе свой водород из свеженькой туманности.
Аноним 29/06/20 Пнд 11:23:25 586133215
>>586127
Наоборот остановить же, чтобы ценное топливо просто так не горело.
Аноним 29/06/20 Пнд 12:48:22 586137216
>>586133
Я говорю про солнечный ветер, который уже выдувает материал из Солнца. При помощи точечного нагрева лазерами этот процесс можно ускорить и магнитными ловушками собирать массу, таким образом на масштабе порядка 10 миллионов лет можно Солнце в красный карлик превратить.
Аноним 29/06/20 Пнд 13:34:45 586139217
>>586137
> Я говорю про солнечный ветер, который уже выдувает материал из Солнца
Это тред тупых вопросов, а не тупых анонов, анон.
Аноним 29/06/20 Пнд 13:53:44 586140218
>>586139
Так что же ты здесь делаешь?
Аноним 29/06/20 Пнд 14:18:41 586141219
Как вообще можно отражать радиоволны от Луны и Венеры? Почему они не поглощаются в грунт Луны и атмосферу Венеры?
Аноним 29/06/20 Пнд 14:21:03 586142220
>>586141
Поглощаются, но далеко не полностью. Тебя ж не удивляет тот факт, что, например, солнечный свет тоже от них отражается, так почему радиоволны должны полностью поглощаться?
Аноним 29/06/20 Пнд 14:46:51 586143221
>>586140
> нет ты!!
А, ну понятно
Аноним 29/06/20 Пнд 15:01:15 586144222
>>586137
>солнечный ветер
Что нам веееетееер, да на это отвееетииит...
Аноним 29/06/20 Пнд 15:06:04 586145223
>>586123
https://ru.wikipedia.org/wiki/Солнце#Солнечное_ядро
>Центральная часть Солнца с радиусом примерно 150—175 тыс. км (то есть 20—25 % от радиуса Солнца), в которой идут термоядерные реакции, называется солнечным ядром[31]. Плотность вещества в ядре составляет примерно 150 000 кг/м³[32] (в 150 раз выше плотности воды и в ~6,6 раз выше плотности самого плотного металла на Земле — осмия), а температура в центре ядра — более 14 млн К.
https://ru.wikipedia.org/wiki/Тройная_гелиевая_реакция
>Тройная гелиевая реакция (тройной альфа-процесс) — цепочка термоядерных реакций, в ходе которой три ядра гелия-4 образуют ядро углерода-12. Она начинается при температуре около 1,5⋅10^8 К и плотности порядка 5⋅10^7 кг/м3.
Как минимум на один порядок.
Аноним 29/06/20 Пнд 17:26:08 586157224
А можно ли послать сигнал с Кьюриосити и принять на Инсайте используя Фобос как рефлектор?
Аноним 30/06/20 Втр 06:38:31 586215225
А откуда гелий на земле, почему он весь не улетел? Водород понятно - связан, а гей же инертный и легкий
Аноним 30/06/20 Втр 10:25:00 586217226
>>586215
Альфа-распады радиоактивных элементов, находящихся в природе.
Торий-232 в своем ряду дает 6 ядер гелия на один атом тория.
Уран-238 дает 10 ядер гелия.
Уран-235 в своем ряду дает 9 ядер гелия.
Аноним 30/06/20 Втр 19:09:30 586292227
>>585265 (OP)
Влияет ли расширение вселенной на уже существующие объекты в плане их размера.
Допустим в максимальной пустоте могучими созданиями создается черная дыра прям вот тютелька в тютельку по массе. Снаружи ставят барьер, чтоб там не долетало ничего.
Вселенная продолжает расширатся и Чд перестанет быть чд?
Аноним 30/06/20 Втр 20:35:38 586339228
>>586292
Во-первых, нет, гравитационно связанные объекты или тем более сами космические тела в размере не увеличиваются, они как бы «проскальзывают» по растягивающемуся пространству. Заметным влияние растяжения становится только на масштабах сверхскоплений галактик, там уже расстояния настолько огромны, что расширение метрики пространства пересиливает гравитацию.

Во-вторых, ты неправильно представляешь себе, что такое чёрная дыра. Это вообще не тело какого-то определенного размера, чёрная дыра это просто точка, там внутри нихуя нет, кроме центральной сингулярности. Горизонт событий это просто расстояние, на котором пространство вокруг сингулярности настолько искривлено, что даже свет начинает проваливаться к центру ЧД, это не какой-то реальный физический объект.

Никакого влияния расширение Вселенной на чёрную дыру не окажет, т.к. размер ЧД определяется только массой центральной сингулярности и больше ничем.
Аноним 30/06/20 Втр 20:49:19 586346229
>>586339
>чёрная дыра это просто точка, там внутри нихуя нет, кроме центральной сингулярности
Не надо прям так уверенно заявлять. Никто толком не знает, что там есть. По ОТО там сингулярность, но ОТО не умеет в квантмех, поэтому нельзя быть стопроцентно уверенным в ее предсказаниях.
Аноним 30/06/20 Втр 21:35:05 586355230
>>586339
Но ведь свет мы видим от галактик которые дальше чем возраст вселенной? То есть летит фотон по плоскости а второй через 1 миллиард лет уже до нас недолетит вроде как, между нами увеличивается?
Тут дело конечно в масштабе, но если вот прям + - 1 кг и черная дыра или не черная дыра и масса не прибавится - ведь в результате расширения пространства ее должно "вывернуть"? Я думаю не мне первому такие мысли приходят, кто-нибудь умный не высказывался?
Аноним 30/06/20 Втр 22:00:47 586359231
>>586355
>Но ведь свет мы видим от галактик которые дальше чем возраст вселенной?
Они теперь дальше. Пока свет от них летел, пространство расширялось.
Аноним 30/06/20 Втр 22:01:37 586360232
>>586355
>Тут дело конечно в масштабе, но если вот прям + - 1 кг и черная дыра или не черная дыра и масса не прибавится - ведь в результате расширения пространства ее должно "вывернуть"?

Чёрная дыра может быть любой массы, хоть в 1 килограмм, тут дело не в масштабе. Если материю уже сжало в сингулярность, обратно она уже не разожмётся, это необратимый процесс.
Аноним 30/06/20 Втр 22:02:57 586361233
>>586360
Хокинг щас перевернулся в гробу.
Аноним 30/06/20 Втр 22:10:24 586363234
>>586361
Совершенно другой процесс, падающая материя полностью пережёвана и затёрта.
Аноним 30/06/20 Втр 22:22:37 586368235
>>586361
ЧД дойдет до планковских размеров, в теории, а вот что дальше - никому не ведомо.
Аноним 01/07/20 Срд 03:48:24 586425236
Итак, готов костюм для выдры. Ищем того, кто сможет запустить. В принципе - на полет 30 минут + 30 минут не так много денег нужно. Скорей всего, выдра вполне спокойно выживет и у меня будет первая в мире космическая выдра
Аноним 01/07/20 Срд 14:24:00 586454237
>>586425
Выдромучитель, гринписа на тебя нет!
Аноним 01/07/20 Срд 14:37:13 586455238
Магнитное поле у Земли есть благодаря Луне и её вращению с получающимся магнитным динамо. Можно ли создать магнитное поле и у луны?
А что если бы у нас вместо Луны был бы Меркурий? Было бы у нас более сильное магнитное поле?
А если подогнать Меркурий в Венере у них получится достаточное по силе магнитное поле, чтобы и Венеру и прикрыть и Меркурий активировать?
Аноним 01/07/20 Срд 14:46:36 586457239
>>586454
Моя выдра будет героем! Запуск через пол годика +-
Аноним 01/07/20 Срд 14:49:30 586458240
15923256008340.jpg 13Кб, 200x382
200x382
>>586455
>Магнитное поле у Земли есть благодаря Луне и её вращению с получающимся магнитным динамо
Аноним 01/07/20 Срд 14:55:20 586459241
Хочу послушать ваше авторитетное мнение о проекте Lunar Gateway. Насколько перспективным он кажется вам? Взлетит или попил бабла? Насколько он является целесообразным? Критики проекта пишут, что эта станция никому не нужна. И почему еще нет треда по этой теме?
Аноним 01/07/20 Срд 15:07:36 586460242
>>586459
>Насколько перспективным он кажется вам?
Нужен для того чтобы репетировать долгое пребывание в космосе за пределами защиты магнитосферы земли, которое неизбежно при полётах к Марсу или другим целям в далеком космосе. Так что рано или поздно врата взлетят, может и под другим названием.
Многие уже получили контракты для работы над этими вратами - СпейсХ получила контракт на логистику и разрабатывает корабль грузовик для полётов к Луне, разрабатывают модули, которые в огромной пачке запихнут на ФальконХэви. Разрабатывают канадскую роборуку. Планируют запуск спутника Капстоун, который проверит как оперировать в ангельской ореольной орбите. НАСА и ЕКА уже разработали пакеты экспериментов для присоединения к вратам.
https://blogs.esa.int/exploration/artemis-introducing-ersa-european-experiment-to-monitor-radiation-in-deep-space/

> Критики проекта пишут
Это не критики, а просто хуесосы разных сортов. Один сорт хуесосов это зубрин петух, которому похуй на Луну вообще и на всё что вокруг Луны хотят делать. Другой сорт это скептики, которые не верят что НАСА вообще что-то может, потому даже когда НАСА говорит "мы это сделаем" скептики ноют, что НАСА сказала это несерьёзно и скоро они передумают.

>почему еще нет треда по этой теме?
Потому что станция еще не полетела и кто-то сомневается полетит ли она вообще. А для обсуждения Луны есть лунатред, он в бамплимите, но я перекачу его через пару минут.
Аноним 01/07/20 Срд 15:13:12 586461243
Аноним 01/07/20 Срд 15:35:46 586465244
>>586461
Все не так, Луна тут вообще никакого отношения не имеет
Аноним 01/07/20 Срд 15:36:44 586466245
>>586461
Фильм " земное ядро " - там конечно фантастика но про магнитное поле все ок.
Аноним 01/07/20 Срд 15:39:31 586467246
Можно ли хрюкнуть в космосе?
Аноним 01/07/20 Срд 16:55:46 586480247
>>586465
Попизди мне тут ещё. Луна с её приливными силами — один из источников разогрева и поддержания в жидком состоянии внешнего ядра Земли, без чего магнитное поле не смогло бы генерироваться.
Аноним 01/07/20 Срд 16:58:26 586481248
>>586467
Ты так говоришь, словно биосфера Земли со всеми её свиньями и не только находится вне космоса.
Аноним 01/07/20 Срд 17:04:30 586483249
>>586480
Это ты сам придумал
01/07/20 Срд 18:06:22 586487250
>>586480
Ядро земли сварено из сыра
Аноним 01/07/20 Срд 18:22:02 586489251
>>586487
Как раз из сыра всегда была Луна
Аноним 01/07/20 Срд 18:48:59 586491252
>>586489
Это устаревшие сведения.
Аноним 01/07/20 Срд 19:33:32 586505253
>>586467
Если бы кое-кто осилили собственную космическую программу, то вполне смогли бы.
Аноним 01/07/20 Срд 20:22:58 586520254
Стикер 63Кб, 250x250
250x250
>>586339
>это не какой-то реальный физический объект.
Тащемта если достаточно глубоко заглянуть, реальные физические объекты становятся еще более ебанутыми, ЧД в этом плане весьма простая.
Аноним 01/07/20 Срд 20:26:52 586522255
image.png 87Кб, 256x256
256x256
>>586455
Платина.
У луны уже не создашь, как и у винуса, они слишком подостыли. Это когда еще внутри бурлит и крутится - что-то получается, а как застывает - уже все.
По-двачерски подскажу. Ты в мамкином супе можешь сделать воронку крутя его поварешкой, и сможешь поддерживать ее без проблем, пока мамка старым полотенцем не отхлестает. А вот без тебя-луняша, батя сделал свой суп, и в нем ты водоворот не сделаешь, как ни крути, будут только обои отслаиваться.
Аноним 01/07/20 Срд 20:29:29 586523256
>>586467
Ближайшее к этому совершили китайцы, завезя на орбиту 40 грамм отборнейшей свиной спермы https://www.airspacemag.com/daily-planet/shenzhous-pigs-in-space-8498500/
Видать, хотели особо нажористых свиней так вывести, или новый энергетический напиток для космонавтов.
Аноним 01/07/20 Срд 20:30:00 586524257
>>586455
>если подогнать Меркурий в Венере у них получится достаточное по силе магнитное поле,
Есть мнение что так и было раньше, но в результате какой-то катастрофы Меркурий перешёл на собственную орбиту. Это объясняет и избыточную плотность Меркурия, и проебанный момент инерции у Венеры, и ее молодую поверхность, а может быть и ее температуру.
Аноним 01/07/20 Срд 20:31:30 586525258
>>586524
>Есть мнение что так и было раньше
А щас ведь хуй проверишь, вообще невозможно узнать, да?
Аноним 01/07/20 Срд 20:39:23 586527259
>>586522
>винуса, они слишком подостыли
Мы нихрена не знаем что у неё внутри, только предложения.
01/07/20 Срд 20:41:13 586528260
>>586527
Знаем - кусок камня
Аноним 01/07/20 Срд 20:41:21 586529261
>>586525
На сегодняшний момент никак. Поскольку никаких миссий ни к Меркурию, ни к Венере в обозримое время не будет, то скорее всего и не узнаем.
Аноним 01/07/20 Срд 20:48:35 586531262
>>586528
/spc на острие науки? Вопрос вулканизма ещё не закрыт, а спейсас уже знает что Вернера- камень.
Аноним 01/07/20 Срд 20:50:39 586532263
>>586527
А, ну венера горяченькая под шубкой, может и не настолько подостыла.
А вот у Марса глазурь такая, что до нуги и не добраться.
Аноним 01/07/20 Срд 21:09:35 586533264
01/07/20 Срд 21:11:51 586534265
>>586531
>ещё не закрыт
Закрыт.
Аноним 01/07/20 Срд 21:14:37 586535266
>>586534
>полтора мимокрокодивших зонда не увидели прям пукающих вулканов
>вопрос закрыт
Нахуя мы зонды на Марс в поисках воды посылали? Можно же было так же вопрос закрыть.
Аноним 01/07/20 Срд 21:16:08 586536267
>>586535
Можно, подповерхностные океаны везде есть же
Аноним 01/07/20 Срд 21:18:05 586537268
>>586536
>подповерхностные океаны везде есть же
Пробурим Луну и купаться будем, да?
01/07/20 Срд 21:34:30 586540269
>>586535
>зонды на Марс в поисках воды посылали?
Только в павильонах Голливуда и Мосфильма. Все эти зонды и выделенные на них деньги - просто попилы на уровне государств.
Аноним 01/07/20 Срд 21:35:07 586541270
>>586540
Сетипетух, ты ли это?
01/07/20 Срд 21:39:29 586542271
>>586541
>Сетипетух
Мне все давно понятно - только у стран, где есть какие то киномощности, внезапно есть и "полеты, зонды, космонавты". Нет киномощностей - нет и "полетов", вот и вся программа по космосу.
Аноним 01/07/20 Срд 21:42:16 586543272
image.png 2Кб, 163x46
163x46
>>586542
Ну а мне понятно, куда летишь ты.
01/07/20 Срд 21:52:17 586544273
>>586543
Вот! Так и затыкают правду.
Аноним 01/07/20 Срд 21:54:50 586546274
01/07/20 Срд 21:55:34 586547275
>>586546
Земля плоская и довольно тонкая, под землей только три слона и черепаха
Аноним 01/07/20 Срд 21:57:07 586548276
>>586546
Аквиферов полно тащемта, притом много слоев. Про артезианскую воду слышал?
Тащемта один из маняпланов по добыче водички на марсе это и есть прокопать артезианскую и там будет химически ебическая вода в жидком виде.
Аноним 01/07/20 Срд 21:57:28 586549277
>>586547
Так были ведь киты, не? А они ведь в океане.
Аноним 01/07/20 Срд 21:59:19 586551278
image.png 461Кб, 548x640
548x640
>>586549
[Смеётся из загробного мира]
Аноним 01/07/20 Срд 21:59:28 586552279
>>586548
Но ведь там, по сути, порода, пропитанная водой. Мокрый песочек, типа.
01/07/20 Срд 22:01:20 586553280
>>586549
>в океане.
в варпе
Аноним 01/07/20 Срд 22:01:37 586554281
>>586552
Есть слои-хуи которые воду не пропускают, глины, скальные породы.
Потому нельзя просто так у нас артезианки копать - воды отличаются от грунтовых, можешь зашкварить и потравить большое количество человеческих жертв, сесть на сгуху и попасть на бабки.
Аноним 01/07/20 Срд 22:03:13 586555282
>>586551
Так и у него черепаха морская, если что. И если википедия не пиздит.
01/07/20 Срд 22:03:31 586556283
>>586554
>просто так у нас артезианки копать
мне заплатили - я выкопал. О том что вода питьевая - не подписывался. Вода только для полива
01/07/20 Срд 22:04:14 586557284
>>586551
это нил армстронг ?
Аноним 01/07/20 Срд 22:05:09 586559285
>>586554
Ну да, поэтому вода из слоев над ними не просачивается вниз. Но ведь это все равно не слой воды (в большинстве случаев, как я понимаю), а слой относительно пористой породы, в трещинах или зазорах между частицами которой сидит вода.
Аноним 01/07/20 Срд 22:12:08 586560286
Аноним 01/07/20 Срд 22:12:11 586561287
>>586559
Я думаю, что с определенной глубины идет вечная мерзлота у этих водоносных горизонтов, которая и ограничивает просачивание и рассеивание, а в жидком виде вода на очень большой глубине где тепло и давление позволяют таковой оставаться.
По крайней мере, такое стоит проверить.
Я считаю ареологию очень перспективной и полезной наукой, даже больше, чем поиск жизни на марсе.
Аноним 01/07/20 Срд 22:14:19 586562288
>>586557
Сэр Теренс Дэвид Джон Пратчетт, единственный человек нормально логично описавший плоскую планету, к модели которой нет претензий даже у самых воинственных апологетов "официальной" науки.
01/07/20 Срд 22:16:25 586563289
>>586562
>единственный человек нормально логично описавший плоскую планету
О! Я понял! Это как Паук отлично описал искуственный интелект в своей песне "Компьютер - гитлер"
Аноним 01/07/20 Срд 22:33:03 586565290
>>586546
Да, недавно нашли на полюсе
Аноним 01/07/20 Срд 23:35:14 586578291
Анон, скажи пожалуйста, а какие агенства мониторят Гелиосферу на предмет вторжения инопланетных объектов (искусственных и естественных) ?
Аноним 01/07/20 Срд 23:37:46 586579292
Аноним 01/07/20 Срд 23:40:06 586581293
Аноним 01/07/20 Срд 23:42:51 586583294
>>586505
Но ведь те кто может издавать такие звуки и так до последнего времени единственными таксистами были.
Аноним 01/07/20 Срд 23:45:28 586584295
>>586579
ПРО разве не может их обнаружить на входе в атмосферу?
Аноним 01/07/20 Срд 23:46:53 586585296
>>586581
Нужды нет.

>>586584
ПРО не следит за объектами у Солнца, смотрят только на землю в ожидании запуска и за объектами на НОО.
Аноним 02/07/20 Чтв 00:03:39 586588297
>>586585
простите, мы видимо друг друга не поняли, Гелиосфера подразумевалось всё то пространство солнечной системы до границ "паузы" воздействия солнечного "ветра" (излучения). А не слежка за солнцем и ближайшей орбиты вокруг солнца.
Аноним 02/07/20 Чтв 00:19:28 586590298
>>586556
>мне заплатили - я выкопал
Рассуждения уровня «мне заплатили — я вылил канистру неизвестной жидкости в городской водозабор». Естественно, подобная отмазка от бутылки не спасает ровно никак.

Без специального разрешения бурить глубже первого водоносного слоя полностью запрещено, неважно для каких целей.
02/07/20 Чтв 00:22:40 586594299
>>586590
Нал из рук в руки без договора - доказывай что я бурил именно на 70 метров, а не то что я пробурил 3 метра а потом ты с дружками добуривал.
Аноним 02/07/20 Чтв 00:28:18 586595300
>>586561
>Я думаю, что с определенной глубины идет вечная мерзлота у этих водоносных горизонтов
Думаешь ты точно не мозгом.
Пиздец, как ты такой тупой живёшь?
Аноним 02/07/20 Чтв 00:30:20 586596301
>>586584
>ПРО разве не может их обнаружить на входе в атмосферу?
Спутники слежения за запуском ракет противника, смотрят на землю, а не в космос.
Блять, откуда вы такие тупари берётесь нахуй?
Пиздец, мне в такие треды нельзя заходит, я просто охуеваю от уровня тупизны этого скотонаселения.
Аноним 02/07/20 Чтв 00:54:35 586600302
>>586583
Разве что таксистами газа из россии в европу.
Аноним 02/07/20 Чтв 00:55:38 586601303
>>586588
А. Не, государства не следят. Астрономы примерно сами по себе смотрят (за них софт сравнивает фотки) и выискивают новые объекты.
Аноним 02/07/20 Чтв 00:56:52 586602304
>>586594
Так докажут же.
Бур чей? Взят когда и кем? Поехали, объясняйтесь.
Аноним 02/07/20 Чтв 00:57:24 586603305
>>586595
Хорошо ты так себя приложил. Никакой логики, никаких опровержений, просто оскорбление.
Аноним 02/07/20 Чтв 01:02:02 586604306
>>586590
А подобным вообще кто-нибудь занимается?
Аноним 02/07/20 Чтв 01:09:51 586606307
>>586604
Полно, повсюду, даже цены не конские.
Аноним 02/07/20 Чтв 01:16:12 586607308
>>586606
Я имею в виду, кто-нибудь ловит тех, кто бурит незаконно, или всем похуй?
Аноним 02/07/20 Чтв 04:57:32 586612309
А как достигается то что ракета лежит на боку у Союза при старте с Байконура? Блоки же равно заправлены. Стол наклонен?
Аноним 02/07/20 Чтв 08:31:36 586619310
Аноним 02/07/20 Чтв 08:40:21 586620311
>>586619
Загоризонтные же.
Я сам в свое время охуел от технологии, но да, они видят ракетный запуск через половину земного шара.
Аноним 02/07/20 Чтв 13:30:52 586642312
Что произойдет, если направить АМС сквозь кольца Сатурна? Каковы шансы удариться о каменюку? Видно ли сам Сатурн изнутри колец, если смотреть вдоль плоскости экватора?
02/07/20 Чтв 13:43:33 586644313
>>586642
>сам Сатурн
Всего лишь миф, мы не видим дальше луны.
Аноним 02/07/20 Чтв 13:44:41 586645314
Что у вас тут за шизик новый несмешной?
02/07/20 Чтв 13:47:37 586646315
>>586645
вот запущу выдру в космос - вы еще обо мне услышите !
Аноним 02/07/20 Чтв 13:57:43 586647316
12.png 38Кб, 1002x678
1002x678
Кстати, можно колонизировать кольца Сатурна. Че там делать хз (прятаться от вражеских АМС, добывать воду например) но я придумал внутрикольцовый толчкоход, пикрелейтед.
Аноним 02/07/20 Чтв 17:31:31 586682317
image.png 377Кб, 2090x1282
2090x1282
>>586642
Очень малы шансы вредно удариться. Кольца имеют хорошее альбедо, но на деле это песок и льдинки и только очень иногда глыбы.
Тем не менее мы говорим об астрономических скоростях и пролетах, так что шансы на фатальное столкновение велики, ответ на твой изначальный вопрос - АМС убьется либо ударом о крупный камень на орбитальной скорости, либо эродирует песком и тоже убьется.
This kills the crab.jpg.
Поэтому Касиня летал в обход основных колец, пикрелейтед.
Аноним 02/07/20 Чтв 20:25:26 586693318
Screenshot20200[...].jpg 409Кб, 720x1520
720x1520
Я правильно понимаю, что черные дыры существуют только из-за того, что скорость света - максимальная во вселенной? Ведь если бы это было не так, то обьект, пересекший Г.С., просто ускорился выше скорости света, достиг перицентра и потом снова замедлился до своей скорости. А из-за этого тупого ограничения ему ничего не остается кроме как падать в Ч.Д.

Я правильно понимаю, что радиус Ч.Д. это ее горизонт событий, а на самом деле вся масса сосредоточена в одной точке сингулярности?

Алсо, поясните плез что на этом пике написано. Я то думал, что Г.С. это просто высота орбиты вокруг Ч.Д, скорость на которой равна скорости света. А тут какую-то дичь про гиперпоаерхности втирают. Да еще и Г.С. есть в прошлом и будущем.
Аноним 02/07/20 Чтв 20:33:48 586696319
>>586693
>Я правильно понимаю, что черные дыры существуют только из-за того, что скорость света - максимальная во вселенной?
Да, черная дыра - это область пространства-времени ограниченная поверхностью на которой скорость убегания равна скорости света.
>Ведь если бы это было не так, то обьект, пересекший Г.С., просто ускорился выше скорости света, достиг перицентра и потом снова замедлился до своей скорости.
Ты превратно понимаешь "скорость света". Это не скорость конкретно фотонов. Это фундаментальная особенность пространства-времени, именно с этой скоростью распространяются фундаментальные взаимодействия. Будь она другой - черные дыры бы были другими, например.
"Скорость света" это просто название для предела скорости.
>Я правильно понимаю, что радиус Ч.Д. это ее горизонт событий, а на самом деле вся масса сосредоточена в одной точке сингулярности?
Никто не знает точно, но скорее всего это так.
>Алсо, поясните плез что на этом пике написано. Я то думал, что Г.С. это просто высота орбиты вокруг Ч.Д, скорость на которой равна скорости света. А тут какую-то дичь про гиперпоаерхности втирают. Да еще и Г.С. есть в прошлом и будущем.
Ты конкретней вопросы задавай, что точно тебе разжевать, а не "мненипанятна".
ЧД это именно область, а не какая-то конкретная вещь.
Аноним 02/07/20 Чтв 21:32:39 586704320
>>585693
Кто вообще решил что спин можно поменять? Нет ли такого что просто спин задан с самого начала, и поменяв спин одной частицы с другой ничего не произойдет.
Аноним 02/07/20 Чтв 21:46:28 586705321
>>586704
>Нет ли такого что просто спин задан с самого начала

Эйнштейн, ты? Хули тебе в гробу не лежится?

Локальные скрытые параметры полностью опровергнуты Беллом и его неравенствами, эксперименты чётко показывают, что до измерения никакого конкретного состояния у частицы вообще нет. Вероятностная волновая функция это не просто отражение нашего незнания истинного состояния системы в реальности, это и есть реальность.

Классические механистические объяснения в квантовой физике тупо не работают, буквально другой результат в экспериментах бы получался.
Аноним 02/07/20 Чтв 21:48:13 586706322
>>586705
Как оно на самом деле работает-то? Все идет по прямой просто так?
Аноним 02/07/20 Чтв 21:50:24 586707323
>>585741
Да тут все ясно. Во второй. Если температура окружающей среды выше или равна 47 градусам. То во второй смесь будет еще нагреваться. Если ниже то все равно во второй, потому что более горячая чашка будет терять температуру быстрее. Можно еще площадь чашки посчитать, но думаю этим можно пренебречь.
Аноним 02/07/20 Чтв 21:53:32 586708324
>>586704
Шляпа слетит только.
Аноним 02/07/20 Чтв 23:03:32 586712325
Бамп
Аноним 03/07/20 Птн 09:41:24 586739326
>>586693
> а на самом деле вся масса сосредоточена в одной точке сингулярности?
Никто не знает, как там на самом деле, ибо существующие теории сингулярность корректно не описывают. Может там шайтаны летают.
Аноним 03/07/20 Птн 12:16:11 586749327
>>586739
Ну вот, что ты наделал?
Представил себе эту чайхану. Сидят степенные шайтаны-акя и пьют чай. Юные шайтанчики возятся в песке. В кустах арчи шумят майнушки. Ошпаз готовится подавать плов...
Аноним 03/07/20 Птн 14:43:33 586761328
Может ли черная дыра разорвать нейтронную звезду приливными силами, высвободить нейтрониум, а потом как-нибудь выкинуть на высокую орбиту, где из него образуется маленькая планета, или где оно просто будет плавать?
03/07/20 Птн 17:23:29 586776329
>>586761
>из него образуется маленькая планета
Нейтрониум без ебической гравитации просто взорвется. То есть, вещество можно из НЗ как-то выбросить, но только вот после этого оно перестанет быть нейтронным.
Аноним 03/07/20 Птн 17:36:45 586780330
>>586776
Если точнее, то не без гравитации, а без давления. Просто в норме гравитация как раз это давление и создает.
Аноним 03/07/20 Птн 18:08:33 586783331
>>586780
Получается если нейтронку чутка разорвать, ее полностью распидорасит?
Аноним 03/07/20 Птн 18:34:27 586785332
>>586783
Ну не так, чтобы чутка, но в принципе да. Гугли распад нейтрона, например. Период полураспада в норме всего 9 минут, то есть он охуенно радиоактивный, и распадается на протон и электрон - составляющие обычной материи. Они не будут без давления сидеть плотно прижатыми, пушо электрически заряженные.
Аноним 03/07/20 Птн 18:39:46 586787333
>>586785
Без гроветации которая бы удерживала дегенератов на месте нейтронка бы стала очень мощно ГОРЕТЬ этим самым распадом нейтронов, ящщитаю.
Аноним 03/07/20 Птн 18:55:33 586788334
А бывают естественные источники когерентного излучения? Ккой-нибудь магнетар дохуя облучающий горячий юпитер атмосфера которого вынужденно светится, например?
Аноним 03/07/20 Птн 18:56:42 586789335
Аноним 03/07/20 Птн 18:57:53 586790336
image.png 345Кб, 1280x720
1280x720
>>586789
Ну вот нихуя ж себе, спасибо.
Аноним 03/07/20 Птн 20:12:29 586793337
>>586749
Это тебе не с этими ЕРЖ пить цикорный эрзац-кофе с свекольным мармеладом и есть гречневые котлеты с ломтиком отрубного серого хлеба с, по случаю прихода дорогого гостя, кусочком рапсового спреда.
Аноним 03/07/20 Птн 20:14:01 586794338
205.jpg 30Кб, 542x616
542x616
>>586793
>>586749
Пожалуйста, продолжайте, у вас какая-то интересная история намечается.
Аноним 03/07/20 Птн 22:11:29 586814339
Посоветуйте хороший, годный учебник по астрофизике и астрономии, не слишком древний. Можно и на английском языке
Аноним 04/07/20 Суб 04:35:08 586857340
>>586607
>или всем похуй?
this
Аноним 04/07/20 Суб 04:38:53 586858341
>>586602
Ты блять прикалываешься штоле?
Люди звонят в ментовку, что убивают, приезжайте спасите помогите. Менты в ответ ржут, что "приедем конечно ваш труп описывать". А ты про то что кто-то будет ловить пацанов с бурилкой.

Соседу два года назад на участке приехали двое пацанчиков просверлили скважину. Метр тыща, 32 метра, 32 тыщи, опускай насос качай.
Аноним 04/07/20 Суб 09:10:51 586869342
Вы охуели, псы ебаные? Где информация про гей-парад планет? В какое время мск и в каком направлении его искать?
04/07/20 Суб 13:16:06 586892343
>>586869
Парад будет что то после 2100-ого
04/07/20 Суб 13:18:19 586894344
Итак. Двигатели приехали, костюм готов, запуск выдры будет!
Аноним 04/07/20 Суб 15:31:53 586920345
Допустим я швыряю тарелочки в атмосфере Земли и на теле с гравитацией Земли но без атмосферы. Где тарелочки будут лететь дальше? Вместо тарелочек можно представить ДВД-диски или пластинки.
Аноним 04/07/20 Суб 17:21:14 586945346
>>586920
Недостаточно данных. А может у тебя тарелочки гиперзвуковые, а? А?
Но если это обычные метательные тарелочки и они правильно запущены так, чтобы хорошо планировать - в атмосфере пролетят намного дольше, конечно.
Аноним 04/07/20 Суб 20:06:08 586959347
>>586858
От локации зависит и от рандома.
В каком-нибудь баттхертостане если палок недособирали будут прямо по дворам ездить лицензии на скважины проверять.
Аноним 05/07/20 Вск 16:47:00 587127348
Челы, я короче долго размышлял и придумал теорию про устройство черных дыр, а так же то, как и где кончается вселенная и почему она не бесконечная, если отталкиваться от теории большого взрыва.
Но так как я шизнутый и не хочу, чтобы какой-то горе ученый-хуй моченый спиздил это. А учитывая 21й век и то, как некоторые любят пиздить всё у других - хочу чтобы авторство было за мной, если реально выстрелит.
Куда податься, чтобы свои шизомысли выплеснуть в эссе и при этом чтобы оригинальное авторство было за мной?
Видос на ютюб? Собственный сайт с одной страницей? Где такие шизобольные публикуются?Кроме двача.
Аноним 05/07/20 Вск 17:01:07 587130349
9ygQwFmo-k.jpg 204Кб, 600x738
600x738
Хало, косманы.
Хочу написать книгу с более-менее твердотой на космическую тему. Если быть точным, очень мне захотелось пройтись по теме системы с несколькими звёздами.
Про то, что бы вы хотели бы почитать в плане сюжетов и историй, это мне конечно же интересно. С удовольствием послушааю ваши манямирки, здравые идеи, вопросы, теории, допущения, предположения и даже видения форм жизни и их условия. На основе это легко смогу сделать парочку историй.
Но сперва надо хотя бы базовую твердоту уточнить, ведь так?
Например, берём для начала вводную: сперва учёные обнаружили далёкую звёздочку в сильнейшие по современным представлениям теле и радиоскопы. Назвали её в честь кого-нибудь, например, Гоголя.
Какк дальше весть номенклатуру?
А Гоголя, Б Гоголя и т.д.?
А если у звезды есть планеты, то они будут, скажем, Б Гоголя 1, Б Гоголя 2, Б Гоголя 3 и т.д.?
А если у планеты Б Гоголя 2 есть луны, как их именовать? Б Гоголя 2а, Б Гоголя 2б и т.д.?
А если у луны Б Гоголя 2а есть своя луна, как тогда еёё обозначать?
А ессли две звезды в ссистеме Гоголя вращаются друг вокруг друга близко, скажем В и Г Гогголя, а вокруг них двоих, по далёёёкой орбите вращается ледяная планета, то как её называть, ВГ Гоголя 1?
А если у В и Г Гоголя есть по парочке пускай и маленьких, но планет, то как тогда считать планету, вращающуюся вокруг общего центра масс В и Г Гоголя, если, наппример, В Гоголя имеет 1 маленькую планетку, а Г Гогголя аж 3? Как тогда считать планету в паре ВГ Гогголя, ту самую далёёкую леедяную?
А может ли быть вообще самая-самая крайня планета, вращаюющаяся вокруг центра масс всех четырёк звёзд системы Гоголя и находящаясятак далеко, что уже почти вышла за пределы гравитационного влияния системы Гоголя?
Оччеь интересно всё это послушать и почитать.

И да, можно предположить, что планета из "Игры Престолов" вращается по переменной траектории между некой обычной звездой и каким-то тёмным карликом, невидимой невооружённым глазом, что может быть че-то от простенькой ЧД, до очёнь чёрной планеты и даже черного или коричневого карлика, что приводит меня к последнему умозрительному вопросу ИТТ - как классифицируются субзвёзды?
Аноним 05/07/20 Вск 19:37:49 587181350
Аноним 05/07/20 Вск 20:02:09 587186351
>>587127
Сумасшедшие фрики обычно в журналах РАЕН публикуются, там и торсионные поля, и пирамиды из бетона, и разумная вода, твоей статье там самое место будет.
05/07/20 Вск 20:36:15 587192352
>>585858
тыж вкурсе что на антивеществе работать не будет двигатель
Аноним 05/07/20 Вск 21:25:51 587208353
Стикер 191Кб, 500x500
500x500
>>587192
Ну антидвигатель.
Аноним 05/07/20 Вск 22:47:55 587237354
>>587208
Который будет тормозить, охлаждаться и излучать тьму?
Аноним 06/07/20 Пнд 17:08:22 587286355
Почему бета-излучение может быть как-то опасно? Ну зарядит тебя до какой-то степени, потом из-за статики вообще не будет долетать. Током стукнет о дверную ручку и ладно.
Аноним 06/07/20 Пнд 17:17:53 587287356
>>587286
Почему поток котлет на скорости 10000 км/ч может быть как-то опасен? Ну наешься ты в какой-то степени, потом из-за обжорства вообще воротить будет. Просрешься потом и ладно.
Аноним 06/07/20 Пнд 17:23:49 587288357
>>587286
Ты путаешь обычную электризацию расчёски об кота с ионизирующим излучением, способным распидорасить воздух с образованием атомарного кислорода, да и из других веществ наделать всяких весёлых радикалов.
Аноним 06/07/20 Пнд 17:30:37 587292358
image.png 556Кб, 540x726
540x726
О, раз уж про радиацию речь пошла, помогите разобраться с этими радами, греями, рентгенами и зивертами?
Я только беккерель и кюри понимаю, но они не имеют прямое отношение к облучению, беккерель это ебаный герц, а кюри это 10 гигагерц, и ниибет, тупо количество распадов, оно уже дальше считается под нужды.
А вот про болезнетворные живительные эффекты на мясо и железо - как оно все понимается? А то читаю и вижу фигу. То джоуль на килограм, и это джоуль на килограмм, но они разные.
Аноним 06/07/20 Пнд 18:03:13 587293359
Neutronradiatio[...].gif 17Кб, 482x292
482x292
>>587292
Грей — джоуль на килограмм. Ничего больше, просто поглощенная доза энергии.

Зиверт — биологический эквивалент джоуля на килограмм, т.е. такое количество радиации, которое надо похавать, чтобы причинить телу тот же ущерб, который бы причинил бы 1 джоуль на килограмм гамма-излучения. Рассчитывается умножением поглощенной энергии в греях на поправочный коэффициент, зависящий от природы радиации. Для альфа-частиц и более тяжелых ядер коэффициент равен 20 (т.е. 1 джоуль альфы в 20 раз опаснее джоуля гаммы), для протонов 2, для электронов 1, для нейтронов целая формула, которая даёт графики как на картинке.

Рад — 1/100 грея. Вышел из активного употребления.
Бэр — 1/100 зиверта. Вышел из активного употребления.

Рентген — более сложная вещь, это такая доза рентгеновского и гамма-излучения, при котором в 1 кг воздуха ионизируется зарядов на ~1/3876 кулона. Устаревшая хуйня без особого физического смысла, применялась только потому, что измерять ионизацию воздуха очень просто, а настоящую поглощенную дозу сложно, поэтому градуированные в рентгенах детекторы радиации были раньше широко распространены. Вышел из активного употребления.

Реально полезных единиц измерения всего две, это беккерель (как сильно фонит источник), и зиверт (как вредно для человека находиться рядом с ним). Остальные единицы это просто архаизмы, которые давно пора забыть нахуй.
Аноним 06/07/20 Пнд 18:07:41 587294360
image.png 230Кб, 400x350
400x350
>>587293
Реально 2 только нужны?
Пикрелейтед.

А про свет можно так же? Люмены, люксы, канделы, ниты?

Блжад, как же с джоулями и ньютонами все просто и понятно, а с излучением ебань начинается.
Аноним 06/07/20 Пнд 19:43:14 587308361
>>587293
>Реально полезных единиц измерения всего две, это беккерель (как сильно фонит источник), и зиверт
>измерять ионизацию воздуха очень просто, а настоящую поглощенную дозу сложно, поэтому градуированные в рентгенах детекторы радиации были раньше широко распространены.
Сейчас тупо переградуировали те же счётчики гейгера-мюллера, так что 1 зиверт = 100 рентген.
Аноним 07/07/20 Втр 05:44:53 587342362
>>587293
>беккерель (как сильно фонит источник)
Беккерель это число распадов в секунду, а распады могут быть ОЧЕ разные. Вплоть до безопасного рутения с его десятком килоэлектронвольт, меньше чем от кинескопа.
Аноним 07/07/20 Втр 05:49:54 587343363
TKs6NdhuVQ.jpg 47Кб, 320x240
320x240
>>587130
>можно предположить, что планета из "Игры Престолов" вращается по переменной траектории между некой обычной звездой и каким-то тёмным карликом, невидимой невооружённым глазом
На эту тему "Задача трёх тел" Лю Цысиня. Коротко - с людьми и даже млекопитающими не взлетит - нельзя предсказать смену сезонов.
Аноним 07/07/20 Втр 15:51:46 587409364
>>587342
>безопасного рутения
Горжусь Россией!
Аноним 07/07/20 Втр 15:52:59 587410365
Стикер 63Кб, 512x285
512x285
>>587409
Подожди, а как же СВИНЕЦ?
Аноним 07/07/20 Втр 18:41:27 587434366
Слушал передачу, и там один астроном говорил, что сейчас уже в теории вполне можно создавать зонд, который будет лететь с 1/4 скорости света. То есть он долетит до соседней звездной системы всего за 16 лет.
И я вдруг осознал, что колоссальность космических расстояний сильно преувеличена. Если наша цивилизация существует всего 4 тысячи лет, и мы уже можем создавать такой аппарат, который от одного конца галактики может долететь до другого всего за 400 000 лет, то для цивилизации, которая существует скажем 100 000 лет вообще не будет никакой проблемы открывать планеты скажем с водой, посылать туда зонды с нанороботами, генетическими кодами, которые будут разворачивать на планетах генетические лаборатории, базы, колонии, и всё это будет происходить в абсолютно божеские сроки, и весь процесс будет автоматизированно воспроизводиться и зонды будут с тех планет отправляться дальше и дальше.
И если уже сейчас 1 гипотетический зонд может пролететь галактику всего за 400 000 лет, то такая цивилизация практически наверняка заселит вообще всю галактику за несколько миллионов лет. Ну пусть 10 миллионов лет, но это даже в пределах существования одного вида вполне приемлемый отрезок времени.
Аноним 07/07/20 Втр 18:44:25 587435367
>>587434
Примерно так почувствовал?
Аноним 07/07/20 Втр 18:46:11 587436368
>>587434
а нахуй это нужно вообще?
Аноним 07/07/20 Втр 18:46:29 587437369
>>587434
>уже в теории
Именно что теории, а не практики.

>наверняка заселит вообще всю галактику за несколько миллионов лет.
Уже придумали, известно как зонд фон Неймана.
Аноним 07/07/20 Втр 18:50:46 587438370
>>587436
Ну это просто свойство жизни. Я понял вдруг, что в этом нет никакой фантастики и полное заселение галактики одним видом, при чем не какими-то абстрактными ИИ, а именно биологическим видом вполне реально в весьма короткие сроки.
Аноним 07/07/20 Втр 18:55:14 587439371
>>587437
>Именно что теории, а не практики.
А какая разница? Там ограничений никаких фундаментальных нет, просто я так понял расчеты какие-то сложные, но это именно уже не фантастика, а реальность. Зонд маленький и к нему будет прилагаться большой парус. Это просто будет реализовано через некоторое время, но факт в том, что в этом нет никаких проблем с точки зрения теории. А значит с развитием технологий и практических не будет проблем.
Аноним 07/07/20 Втр 19:05:31 587442372
>>587439
>ограничений никаких фундаментальных нет
каждый раз когда я встречаю такое выражение далее идет отборный бред
Аноним 07/07/20 Втр 19:07:24 587443373
>>587442
Это же не я говорю, а астроном, забыл его фамилию, он часто выступает. Ему наверно виднее.
Аноним 07/07/20 Втр 19:11:42 587444374
>>587439
>я так понял расчеты какие-то сложные
>Зонд маленький и к нему будет прилагаться большой парус.
Ты про "лазерную тягу"? Если так, то одна из проблем будет в том что лазерный луч будет расходиться слишком быстро. Это основы оптики, там те же принципы что формируют диаграмму направленности у параболических антенн или ограничивают разрешающую способность телескопов. И множество других сложностей, увеличивающих затраты на подобное до бесконечности.

>А какая разница?
Разница в том, что когда это станет возможно практически, наверняка все уже изменится, возможно включая теорию.
Аноним 07/07/20 Втр 19:17:38 587448375
>>587444
>наверняка все уже изменится
Наверняка в лучшую сторону. Енивей явно существует такой простой способ экспансии, и явно высокоразвитая цивилизация, прошаренная в робототехнике и генной инженирии будет способна его реализовать, и заселить без всяких варп-двигателей всю галактику в разумные сроки.
Аноним 07/07/20 Втр 19:31:12 587449376
>>587448
>явно высокоразвитая цивилизация, прошаренная в робототехнике и генной инженирии будет способна его реализовать, и заселить без всяких варп-двигателей всю галактику в разумные сроки
Но этого не произошло.
Все потому что хуманы телескопом своим воюют не в ту сторону.
Потому что надо не вверх, а вдоль внутрь.
Аноним 07/07/20 Втр 19:46:34 587452377
>>587343
Читал. Но там, объективно говоря, задача четырёх тел, ведь мы рассматриваем планету цивилизации трисолярцев как тоже тело, а это +1 к тройной звезде
Аноним 07/07/20 Втр 19:49:20 587453378
>>587452
Тем более, надо учитывать, что ЗТТ рассматривается, насколько я помню, для тел одинаковой массы и разщмера на примерно равно расстоянии, а в мире гипотетического мира "Игры престолов" естьтолько 11 звезда-светила, один крупный гравитационные незримый объект и сама планета с ИП, крутящаяся вокруг звезды-светила
Аноним 07/07/20 Втр 21:05:27 587484379
1200px-RocheLob[...].png 297Кб, 1200x1253
1200x1253
>>587453
>ЗТТ рассматривается, насколько я помню, для тел одинаковой массы и разщмера на примерно равно расстоянии
Нет, тебе траектории и в системе Земля-Луна прекрасно расколбасит, если будешь лететь слишком медленно в окрестностях точек Лагранжа.
Аноним 07/07/20 Втр 21:20:23 587486380
>>587208
при анигиляции позитрон-електронной пары, происходит рождение гамма квантов, у человечества (и во вселенной) нет зеркал способных отражать гаммма кванты
Аноним 07/07/20 Втр 21:28:37 587487381
>>587486
>електронной
Хiхiл, спок.
07/07/20 Втр 21:31:32 587489382
>>587487
я хихил йобаный - мне можно
Аноним 07/07/20 Втр 23:13:16 587514383
e6537609d8a5bea[...].gif 758Кб, 500x500
500x500
>>587486
>нет зеркал способных отражать гаммма кванты
А и не надо их отражать, надо вывернуть надутый пузырь Альбукерке так, чтобы всё излучение из него ебашило в одну сторону.
Аноним 07/07/20 Втр 23:36:51 587536384
>>587486
>при анигиляции позитрон-електронной пары, происходит рождение гамма квантов, у человечества (и во вселенной) нет зеркал способных отражать гаммма кванты
Во-первых не факт.

Во-вторых гамма-кванты не обязательно отражать, их можно с одной стороны поглощать и конвертировать во что-то полезное. Например, установку по добыче электричества из гамма-квантов с близким к 100% кпд я в прошлых тредах описывал. Это электричество идёт на лазер, тем самым получается функциональный аналог зеркала.
Аноним 08/07/20 Срд 01:21:36 587562385
>>587486
Ну поглощай их хули, тоже импульс.
Аноним 08/07/20 Срд 08:00:06 587576386
>>587449
>Но этого не произошло.
Ну возможного этого не произошло из-за ничтожной вероятности именно зарождения жизни в галактике, и мы будем первыми, кто её заселит, раз это так просто и быстро можно реализовать.
Аноним 08/07/20 Срд 08:21:03 587579387
>>587514
Ты его надул, чтобы выворачивать?
Аноним 08/07/20 Срд 09:29:25 587585388
Не могу понять/представить четырехмерное пространство, пиздос... Памахите! У меня видимо в голове, что-то не работает. На вас вся надежда!
Аноним 08/07/20 Срд 09:40:38 587587389
>>587585
А волшебного средства понять и нет.
Тебе надо много и упорно работать над этим.
Аноним 08/07/20 Срд 09:57:53 587591390
>>587587
Каковы ваши предложения? пойти нахуй
Аноним 08/07/20 Срд 10:35:14 587599391
>>587591
Я же сказал - упорно работать над этим.
Ну если конкретно, то например, разработка собственного 4д рендера отлично поможет.
Аноним 08/07/20 Срд 10:46:05 587608392
VID202006101609[...].mp4 1904Кб, 384x480, 00:00:15
384x480
>>587599
На этом мои стремления в познании паходу всё.
Аноним 08/07/20 Срд 10:50:22 587610393
>>587434
Ещё 26 тысяч лет осталось подождать.
Аноним 08/07/20 Срд 15:16:49 587650394
>>587434
Цивилизация подразумевает связность, иначе это не цивилизация, а кучка изолированных островов с папуасами, где каждый папуас заперт в пределах острова, и всякое сообщение между островами отсутствует.

Мы в любом случае обречены дрочить низкоуровневую природу пространства до тех пор, пока его не порвём и не сделаем сверхсвет. Неделимый атом порвали - значит и пространство порвём. Надо всего лишь хорошо (и в правильное место) стукнуть.
Аноним 08/07/20 Срд 15:22:00 587651395
>>587650
Да, иначе это кучка разных цивилизаций, которые со временем станут разными видами.
Аноним 08/07/20 Срд 15:22:47 587652396
>>587650
Я тебе по голове стукну. Законы вселенной нельзя нарушать, иначе накажут.
Аноним 08/07/20 Срд 15:23:37 587654397
>>587651
>станут разными видами
И чо? Разные виды вполне могут работать вместе как одна цивилизация. Мы же как-то уживаемся с неграми и хохлами)))))
Аноним 08/07/20 Срд 15:23:54 587655398
image.png 158Кб, 1280x720
1280x720
>>587652
Как что-то плохое. Вселенная все равно равно или поздно кончится, так пусть с лулзами.
Аноним 08/07/20 Срд 15:24:45 587656399
image.png 1042Кб, 1438x1080
1438x1080
Аноним 08/07/20 Срд 15:29:33 587658400
>>587655
Ты охуел кончать вселенную - это как огромный корабль на котором 1000 людей в своих каютах сидят, не могут друг с другом поговорить из-за переборок(у нас расстояния), но один пидор решил что он один такой и решил пробить дно корабля, утянув на дно не только себя, но и всех остальных. Тебе бы понравилось если бы сейчас всленная начала исчезать и ты бы даже не понял почему это происходит, а произошло это потому что в 1000000 миллиардах световых лет какая-то цивилизация морских ежей порвала ткань пространства и времени ради лулзов или эксперимента, и таким образом убила вселенную для всех?
Аноним 08/07/20 Срд 15:33:41 587660401
>>587658
Да ладно, не кипятись. В другой вселенной попробуем снова.
Аноним 08/07/20 Срд 15:34:15 587661402
>>587650
Меня просто возбудило осознание, что это реализуемо теоретически в самые короткие сроки без всякого сверхсвета, и при этом не реализовано фактически судя по всему. Хотя галактика существует миллиарды лет.
Аноним 08/07/20 Срд 15:40:36 587662403
>>587660
да я не кипячусь, тот факт что вселенная не умирает и не светится всякими неестественными цветами как раз доказывает, что она почти неразрушима и что разум редок. Планет с тваринами может быть дохера, а там где животные учатся технологиям нихуя. Прям миллионы планет со своими птицами, растениями, рептилиями, динозаврами, которые живут, жуют, срут, но ничего более и так пока их звёзды не дохнут.
Аноним 08/07/20 Срд 15:41:04 587663404
end times.webm 3436Кб, 256x256, 00:01:59
256x256
>>587658
>Тебе бы понравилось если бы сейчас всленная начала исчезать
Да, я не против. Охуенно было бы если бы еще это играло. Я уже смирился с конечностью бытия.
Аноним 08/07/20 Срд 15:47:14 587665405
Был спор по поводу того, входят ли другие туманности в нашу галактику. И одним из аргументов спора заключался в том, что в какой-то момент в туманности наблюдался взрыв сверхновой, который был неправдоподобно ярким, то есть если бы туманность была целой галактикой, то он бы не смог бы перебить её свет.

Я никак не могу найти инфы по этому поводу, как этот взрыв, который закрыл светимость целой галактики объясняется сейчас? Как это возможно?
Аноним 08/07/20 Срд 16:58:19 587689406
001.jpg 399Кб, 809x1106
809x1106
>>587652>>587658
Пикрелейтед. Вселенная кончилась ещё тогда, когда первый безумный Тумба принёс в пещеру факел, которым можно сжечь весь лес!
Аноним 08/07/20 Срд 17:18:24 587699407
>>587665
Сверхновые легко перекрывают яркость всей галактики.
Аноним 08/07/20 Срд 17:36:28 587708408
image.png 665Кб, 652x652
652x652
Аноним 08/07/20 Срд 17:40:03 587709409
А может ли гипотетически работать звезда на реакциях распада, состоящая из тяжелых элементов?
Планету из кротов собирали, что будет, если звезду из природного урана собрать?
Аноним 08/07/20 Срд 18:05:59 587718410
Аноним 08/07/20 Срд 20:21:07 587775411
>>587709
Из природного урана она нахуй взорвётся, ещё не родившись. Даже из 238.

Нужен какой-то едва фонящий элемент, чья критическая масса соизмерима с массой звезды. И для начала надо этот элемент в товарных количествах и в высокой концентрации где-то взять.

А так-то тлеющие ядерные реакции тяжёлых элементов в недрах планет идут, в какой-то степени это и есть твоя звезда.
Аноним 09/07/20 Чтв 02:36:21 587810412
>>587775
>нахуй взорвётся
Так звёзды и так взрываются же. А гравитация их удерживает.
У наших газовых звезд есть предел массы выше которого гравитация уже не удерживает рассеяние разогретой массы из-за энергетики происходящего.
У звезды из какого-нибудь урана будет уже скорее предел в виде радиуса шварцшильда, если докидывать уран, то с какой-то массы она становится черной дырой. Уран сильно плотнее, а еще менее энергетичный, чтобы урановая плазма слишком далеко разлеталась.
В итоге если посидеть и посчитать можно прикинуть параметры этой искусственной звезды, ее примерный график и жизненный цикл.
Звучит как прикольный мюсленный экскремент. Надо Рэнделлу Манро написать, может поугарает.
Аноним 09/07/20 Чтв 10:05:33 587827413
unlockstephenha[...].png 54Кб, 262x338
262x338
Хокинг был полностью парализован. Сохранялась ли у него чувствительность конечностей и тела? Или его можно было подвесить как пиньяту и пиздить палками, а он бы ничего не почувствовал?
Аноним 09/07/20 Чтв 10:13:39 587830414
>>587827
АЛС задействует только моторную часть коры мозга.
Аноним 09/07/20 Чтв 13:40:53 587855415
Аноним 09/07/20 Чтв 13:47:20 587856416
Аноним 09/07/20 Чтв 14:16:03 587860417
>>587856
Ну просто what-if.xkcd.com не обновляется.
Аноним 09/07/20 Чтв 14:21:08 587861418
hellohtmlm14d5b[...].jpg 127Кб, 880x528
880x528
Итак, предварительный запуск выдры на 8 октября
Аноним 09/07/20 Чтв 14:23:19 587863419
>>587860
Да, он там охуел походу совсем.
Надо его ИРЛ найти и отмудохать со словами "где апдейты вотифа, где, сука?", и сказать, что если не будет хотя бы раз в месяц апдейты постить, то сломаем ноги.
Кто возьмется за такую задачу?
Аноним 09/07/20 Чтв 17:03:13 587889420
w8GizO4WShw.jpg 654Кб, 1261x1711
1261x1711
yz-oYm6lTnE.jpg 421Кб, 1005x1366
1005x1366
Возможно ли по аналогии с воздушными шарами в атмосфере Венеры "Вега-1 и 2" запустить в атмосферу Юпитера зонд на воздушном шаре? Возможен ли полет воздушного шара в разреженной атмосфере Марса?
Аноним 09/07/20 Чтв 17:13:02 587890421
>>587889
На юпитере ветра ебические жи.
А на марсе хуй, можешь попробовать жидким вакуумом заправить.

Сверхлегкий вертолет зато будет летать, да, физика позволяет.
Аноним 09/07/20 Чтв 17:14:19 587891422
>>587889
на юпитере нельзя так как там атмосфера водород, а легче него ничего нет
на марсе можно в теории, но очень невыгодно
Аноним 09/07/20 Чтв 17:26:15 587893423
>>587889
В теории может летать монгольфьер с нагретым водородом. На практике же он должен быть очень большим (т.к. g большое), да и просто его страшным ураганом распидорит к хуям.

Также может летать аппарат тяжелее воздуха, а энергию брать да хоть из того же градиента ветра, были такие концепты - два планера, связанные тросом, в слоях с разной скоростью. Или тупо планер, ловящий термалки, хуялки или что там есть восходящее, 100% есть. На практике опять же планерам и самолётам тупо пизда, уж слишком буйные процессы прлисходят из-за более сильной гравитации и необъятности дедули Юпа.
Аноним 09/07/20 Чтв 17:32:39 587899424
>>587893
>g большое
Ващет, это строго похуй. Во сколько раз вырос вес нагрузки, во столько же выросла и подъемная сила. Большим он должен быть потому, что вокруг тоже водород, ненамного плотнее того, что внутри.
Аноним 09/07/20 Чтв 17:43:58 587900425
>>587827
Учитывая что он был два раза женат и у него было трое детей, какая-то чувствительность где-то по-люблму оставалась
Аноним 09/07/20 Чтв 17:47:32 587901426
>>587900
Последнего ребенка, Тимоти, он заделал в 79 году, он только тогда стал вынужден пользоваться креслом.
А для брака рабочий ШТАНГЕНЦИРКУЛЬ не является необходимым. Хотя технически он да, мог работать.
Аноним 09/07/20 Чтв 18:20:59 587905427
Мальчик, музыка ...
Во-первых, Алькубье.
Во-вторых, он не падает, но вы строите ковчег изнутри (иначе вы не родитесь).
В-третьих, НАСА управляло тысячами самолетов
Это точка доступа для больших проектов, как в прошлом.
Рак Антарктиды.
Аноним 09/07/20 Чтв 18:22:51 587906428
thinking emoji [...].gif 455Кб, 256x256
256x256
>>587905
>вы строите ковчег изнутри (иначе вы не родитесь)
Как проницательно.
Аноним 09/07/20 Чтв 22:25:41 587944429
15703555978660.jpg 19Кб, 604x453
604x453
>>587650


Пока звезды погаснут времени куча

Может даже удастся сбежать за пределы Вселенной

Ей то пиздец все равно
Аноним 10/07/20 Птн 04:10:55 587960430
>>587944
Надо найти Очко Вселенной, которое старше ее самой и заныкаться в нём.
Аноним 10/07/20 Птн 06:42:33 587965431
Почему у нас большая полуось? Есть малая полуось?
Почему у них полубольшая ось? Есть полумалая ось?
Аноним 10/07/20 Птн 13:38:16 587978432
Аноним 10/07/20 Птн 13:40:20 587979433
>>587978
Бред, это просто среднее значение орбиты, оно одинаковое в любом случае, это перигей поделенный на апогей, неважно что на что делишь - значение половинное одинаково всегда
Аноним 10/07/20 Птн 15:56:07 588000434
Аноним 10/07/20 Птн 18:29:57 588022435
>>587130
>А если у луны Б Гоголя 2а есть своя луна, как тогда еёё обозначать?
не бывает лун с лунами
>А ессли две звезды в ссистеме Гоголя вращаются друг вокруг друга близко, скажем В и Г Гогголя, а вокруг них двоих, по далёёёкой орбите вращается ледяная планета, то как её называть, ВГ Гоголя 1?
не уверен, что на таких расстояних, даже при относительной близости звёзд, возможна общая планета
>А может ли быть вообще самая-самая крайня планета, вращаюющаяся вокруг центра масс всех четырёк звёзд системы Гоголя и находящаясятак далеко, что уже почти вышла за пределы гравитационного влияния системы Гоголя?
это ещё более невероятно
>Про то, что бы вы хотели бы почитать в плане сюжетов и историй, это мне конечно же интересно
поиграв ввов в своё время задумался, как на планете может быть несколько разумных доминирующих видов. напиши и обоснуй, какие у тебя в истрии есть разные виды и обоснуй, почему они там и почему они разные
Аноним 10/07/20 Птн 19:48:17 588036436
На чём работает электроновский Kickstage? На вонючке или керосине?
Аноним 10/07/20 Птн 19:50:52 588037437
>>588036
Говорят про "зеленый" маняпропеллент.
Если не дебилы, будут гидразин юзать, он не зря прошел проверку временем блин.
Аноним 10/07/20 Птн 21:19:57 588053438
>>588037
Может быть, кучерявый пиздит, и под названием экологически чистой хуеты скрывается обычная вонючка?
Аноним 10/07/20 Птн 21:21:25 588054439
>>588053
А хули такого особо грязного в ММГ? Качественные и практичные виды топлива.
Аноним 10/07/20 Птн 21:27:06 588057440
>>588054
Загугли токсичность гидразина и его производных
Аноним 10/07/20 Птн 22:56:59 588073441
>>588037
>>588053
>>588054
Дураки штоле?

>>588036
HAN+ADN, любая смесь с закисью азота, перекись водорода, что угодно может скрываться под этим, пока не раскроют не узнаем. Точно не ALICE, лол.

HAN+ADN звучит как наиболее подходящий вариант, т.к. они по всем параметрам лучше гидразина, кроме масштабирования - не подходят для больших установок. Электроновский кикстейдж совсем маленький, по сути это компактный РБ с компоновкой Фрегата и намного меньшими масштабами, движок на 120 ньютон, там явно что-то с человеческим импульсом, а не перекись. И на криогенику тоже непохоже.
Аноним 10/07/20 Птн 23:01:05 588074442
>>588053
Тогда бы понадобились допуски, оборудование, сертификации, и вообще с гидразином ебли не оберёшься, что сильный перебор для такой мелкоракеты. Ззеленые топлива юзают из-за простоты, в поле заправил, с грузовика запустил.
Аноним 10/07/20 Птн 23:02:44 588075443
>>588022
>не бывает лун с лунами
Ничто не запрещает им существовать же.
Аноним 10/07/20 Птн 23:14:29 588078444
>>588073
Хотя везде пишут что там некая однокомпонентная херня. Значит HAN+ADN и криогеника точно отпадают. Либо смесь какой-нибудь хуйни с закисью, либо перекись и катализатор, либо каталитическое разложение какой-нибудь производной ADN как вариант, например
Аноним 11/07/20 Суб 03:48:12 588102445
>>588057
>токсичность гидразина
Так нехуй его пить, блядь. Его в ракете сжигать надо.
Аноним 11/07/20 Суб 11:16:32 588126446
maxresdefault.jpg 16Кб, 1280x720
1280x720
Аноним 11/07/20 Суб 11:20:29 588129447
>>588126
Если площадку не распидорасило то похую.
Аноним 11/07/20 Суб 16:09:44 588181448
1594472983371.jpg 572Кб, 1671x1086
1671x1086
Как называются эти маленькие крылышки в передней части самолёта?
Аноним 11/07/20 Суб 16:12:23 588182449
Аноним 11/07/20 Суб 16:14:05 588183450
>>588182
Спасибо, но было вроде какое-то русское название
Аноним 11/07/20 Суб 16:18:16 588184451
>>588183
Ты ждешь слова типа "антикрыло" или чего?
Аноним 11/07/20 Суб 17:58:58 588208452
>>588181
Переднее горизонтальное оперение (ПГО)
Аноним 11/07/20 Суб 19:54:24 588221453
000029159431182[...].jpg 93Кб, 870x486
870x486
Местные долбоебы вообще следят за событиями в космосе или нет? Посбивались по шконкам и сидите надрачиваете, пока по небу кометы летают, а планеты проводят парады
Аноним 11/07/20 Суб 20:27:34 588229454
>>588075
назови известные луны сосвоими лунами
Аноним 11/07/20 Суб 20:31:22 588232455
Аноним 11/07/20 Суб 20:32:48 588233456
>>588229
я слышал на сатурне у кого-то есть
Аноним 11/07/20 Суб 20:39:28 588236457
>>588221
Комета хуйня какая-то, едва видимая невооруженным глазом, а парада планет и вовсе не было — там угол между планетами в 100+ градусов.
Аноним 11/07/20 Суб 20:57:04 588241458
>>588232
ага, минмус наверное
>>588233
>я слышал на сатурне у кого-то есть
назови
Аноним 11/07/20 Суб 22:25:45 588255459
>>588241
Вокруг Реи стабильные орбиты могут быть, и там даже подозревали наличие колец, но на фотках с Кассини нихуя не нашли.
Аноним 11/07/20 Суб 23:55:17 588273460
>>588229
Перечитай ещё раз на что отвечаешь.
Аноним 12/07/20 Вск 00:10:39 588275461
3362967wide-e9c[...].jpg 66Кб, 1022x574
1022x574
1567218196884-R[...].jpg 159Кб, 410x290
410x290
171214-D-YE683-[...].JPG 84Кб, 522x400
522x400
86601291151f354[...].jpg 192Кб, 1024x683
1024x683
>>588102
И гидразин и АТ, внезапно, надо синтезировать, перевезти и заправить. Для этого нужны пиздец какие предосторожности, это без пяти минут химическое оружие. Допуски, обучение, спец. оборудование и отдельное здание для заправки с полной фильтрацией, сертификация, трудовые гарантии, специальная подготовка пожарных и т.п. Приближаться к севшему аппарату с вонючкой могут только спецлюди, а любой потенциальный контакт с ним должен быть задокументирован. Если падает истребитель с гидразином в ВСУ, тот же F-16 например, там целая спецоперация по чистке унитаза вилкой идёт, хотя там его хуй да нихуя. С вонючкой столько ебли, что от неё с радостью избавятся, если будет подходящая альтернатива.

>Его в ракете сжигать надо.
Ракетные двигатели не сжигают 100% топлива, они работают с избытком по горючему, выбрасывая определённый процент наружу. Определённое количество вонючки остаётся ещё и в трубах в отработавшем аппарате, актуально для нижних ступеней.
Аноним 12/07/20 Вск 02:22:17 588288462
Вопросы такие. Если на какую-нибудь любительскую ракету петарду поставить электрический ракетный двигатель сопоставимого с этой ракетой размера, на какую высоту сможет подняться ракета?

Можно ли электрический ракетный двигатель поставить на планер? Летать будет?
Аноним 12/07/20 Вск 02:25:17 588289463
>>588288
> Если на какую-нибудь любительскую ракету петарду поставить электрический ракетный двигатель сопоставимого с этой ракетой размера, на какую высоту сможет подняться ракета?
Не оторвется от земли.
>Можно ли электрический ракетный двигатель поставить на планер? Летать будет?
Можно. Будет планировать как и планировал. ЭРД на высоте где-то 250 км способен компенсировать драг атмосферы. Ниже - не способен, его миллиньютоны тяги никакой погоды не играют.
Аноним 12/07/20 Вск 02:38:21 588290464
>>588289
То есть в тропосфере и стратосфере электрический ракетный двигатель бесполезен. Спасибо за ответ.
Аноним 12/07/20 Вск 02:42:35 588291465
>>588290
Он не способен дать заметную тягу при внешнем давлении.
Поэтому их испытывают исключительно в вакуумных камерах.
Аноним 12/07/20 Вск 03:09:50 588292466
>>588289
>миллиньютоны тяги
Теоретически возможны электронагревные двигатели с большой тягой, например электродуговые. Практически для них батареек нет, поэтому исследования почти не ведутся.
Аноним 12/07/20 Вск 03:13:57 588293467
>>588292
Когда ты в атмосфере тебе для движения достаточно задействовать эту самую атмосферу. Например, пропеллером.
Аноним 12/07/20 Вск 03:15:06 588294468
>>588288
>Если на какую-нибудь любительскую ракету петарду поставить электрический ракетный двигатель сопоставимого с этой ракетой размера, на какую высоту сможет подняться ракета?
У ЭРД всё зависит не от размера двигателя, а от количества электроэнергии, которое ты готов приложить. Для понимания масштабов, мгновенная мощность, выдаваемая всего лишь одним ЖРД Merlin на полной тяге - порядка гигаватта, это мощность большой электростанции. Такой источник энергии на ракету не запихать, а если и запихаешь, он быстро разрядится. Плотность хранения электроэнергии намного меньше плотности химических топлив. (не считая ядерных источников, естественно)

Кроме того, при одной и той же мощности ты должен выбирать - удельный импульс или тяга. Если хочешь и то и другое сразу, нужно увеличивать мощность.

>>588290
>>588291
Зависит от типа ЭРД. Ионные двигатели способны работать только в глубоком вакууме. Магнитодинамические ЭРД в принципе могут и при давлении работать, хоть и с низкой эффективностью, т.к. это банально плазменная горелка с микроволновой печью, производящей плазму, и магнитом, её разгоняющим. А тем более электродуговые. МГД-двигатели (не плазменные, разумеется) даже в лодки ставили, ради хохмы эксперимента.

>>588289
ЭРД это далеко не только двигатели Холла, да и они способны выдать ньютоны и больше - знай только питай. На практике даже ионники бывают на десятки киловатт, при этом . Опять же, плазменные движки могут иметь на порядки большую тягу, сравнимую с химическими, и динамически разменивать тягу на импульс по желанию (см например VASIMR)

>>588292
Электродуговые двигатели используют в гиперзвуковых аэродинамических трубах, например.

>>588293
Это уже будет не ракетный двигатель. ты только что Rotary Rocket
Аноним 12/07/20 Вск 03:16:05 588295469
>>588294
>На практике даже ионники бывают на десятки киловатт, при этом выдавая единицы ньютон
фикс
Аноним 12/07/20 Вск 03:18:59 588296470
>>588294
Очень хороший пост, спасибо.
Аноним 12/07/20 Вск 03:24:45 588297471
i.jpg 23Кб, 344x320
344x320
>>588294
>Плотность хранения электроэнергии намного меньше плотности химических топлив.
Это не догма. Так-то аккумуляторы тоже химические источники, и уже подходят к этим пределам. Другое, что если использовать эту энергию для нагрева рабочего тела, получится пикрелейт.
Аноним 12/07/20 Вск 03:26:15 588298472
>>588297
>Так-то аккумуляторы тоже химические источники, и уже подходят к этим пределам
Нет. Одноразовые химические источники энергии по прежнему на порядок менее энергоемкие чем прямое сжигание солярки.
Аноним 12/07/20 Вск 03:27:06 588299473
>>588297
Имеется в виду сколько ты получишь электроэнергии на 1кг аккумуляторов, и сколько тепловой от сжигания 1кг топливной пары. Разница там в районе порядка, если не больше.
Аноним 12/07/20 Вск 03:27:17 588300474
>>588298
>порядок менее энергоемкие чем прямое сжигание солярки.
Окислитель не забыл?
Аноним 12/07/20 Вск 03:30:11 588301475
>>588300
Нет, по прежнему порядок.
Аноним 12/07/20 Вск 03:35:30 588302476
>>588301
Берём водород, кислород, соединяем в топливном элементе, почему на порядок меньше энергии получится чем сжигать? А любой химический источник тока это топливный элемент, только реагенты в твёрдом виде хранятся в основном. Хотя для возможности реакции всё равно их надо растворять в электролите, так что это не догма тоже.
Аноним 12/07/20 Вск 03:37:27 588303477
>>588302
>почему на порядок меньше энергии получится чем сжигать?
Мы говорим про реактивный движитель же.
Тут просто КПД тебе по зубам дает.
Аноним 12/07/20 Вск 03:41:20 588304478
Аноним 12/07/20 Вск 03:42:27 588305479
image.png 12Кб, 552x127
552x127
>>588304
Спизданул как господь.
Аноним 12/07/20 Вск 03:42:54 588306480
>>588302
>Берём водород, кислород, соединяем в топливном элементе, почему на порядок меньше энергии получится чем сжигать?
Так если у тебя водород и кислород уже есть - то проще сразу сжигать, чем в электричество гнать с непонятным КПД, и через тридевять преобразований с лишней массой преобразователей разгонять этим отдельно хранимое рабочее тело.
Аноним 12/07/20 Вск 03:44:03 588307481
231927691764024[...].jpg 27Кб, 720x540
720x540
>>588303
>КПД тебе по зубам дает
Не на порядок. Меньше, конечно, зато гибкость применения выше - электричество вкуснее просто тепла.
Аноним 12/07/20 Вск 03:46:57 588308482
>>588307
Ты сжигая его и выбрасывая воду можешь при этом тепловую энергию в электричество преобразовывать. 50% КПД реактивной тяги + 1% электричества (грубо говоря)
Против 10% электрического КПД который ты хуй знает как задействуешь для тяги.

Нет, анончик, для тяги сжигать топливо это самое эффективное решение если мы говорим про э/м взаимодействие.
Аноним 12/07/20 Вск 03:49:02 588309483
>>588306
Зато появляется возможность весь выхлоп не выбрасывать, а сохранить и потом назад в компоненты перезарядить, как электричество появится.
Аноним 12/07/20 Вск 03:51:22 588310484
>>588308
>10% электрического КПД
Чё-то ты совсем батарейки принизил. По моему опыту, они вовсе даже не теряют 90% при разряде в тепло, и вообще при заряде только греются.
Аноним 12/07/20 Вск 03:52:10 588311485
>>588309
Это уже совсем тролльячья физика. Отброс выхлопа придаёт кинетическую энергию ракете. Собирая его обратно, ты остаёшься без ракеты.
Аноним 12/07/20 Вск 03:55:24 588312486
>>588310
Анон, когда батарея выделяет заложенную в нее энергию по максимуму - ты это видишь.
БОльшая часть энергии не расходутся при "полностью разряженной" батарее, ты по прежнему технически можешь выделить дохуя энергии из нее.
https://youtu.be/aRutyaxmdrs
Видео стронгли рилейтед.
И это наверное единственный вид электрического огня который можно и нужно тушить водой.
Аноним 12/07/20 Вск 04:14:14 588313487
>>588311
Не весь, а часть, конечно. За счёт высокого УИ электродвижков могу позволить, в принципе.
>>588312
Так и в ЖРД не все компоненты сгорают целиком. В водородниках даже подают на 20-30% больше водорода, чем по стехиометрии, чтобы УИ улучшить.
Аноним 12/07/20 Вск 04:42:30 588314488
>>588313
>Так и в ЖРД не все компоненты сгорают целиком. В водородниках даже подают на 20-30% больше водорода, чем по стехиометрии, чтобы УИ улучшить.
А я не утверждал, что ЖРД это 100% КПД.
Ты сейчас никак не опровергаешь мои слова, просто уточняешь.
Даже при таком неэффективном расходе оно выходит эффективно.
Аноним 12/07/20 Вск 09:50:06 588327489
поясните, кто в наса называет марсоходы? они троллят или это специально? сначала были спирит - дух и опотюнити - то есть возможность, потом кюрьёсити - любопытство, теперь персивиренс - упорство/назойливость. это что за путь горе-пикапера такой? возможность - любопытство - назойливость? что дальше будет, как следующий назовут может быть похоть или бесстыдство?
Выдуваем космические корабли Аноним 12/07/20 Вск 12:00:26 588334490
maxresdefault-2.jpg 106Кб, 1280x720
1280x720
Анончики, такая охуительная идея пришла в голову пару минут назад. Когда мы полетим таки к звёздам, нам потребуются OCHE большие корабли, ведь сверхсвета то нет, а топливо со жратвой нужно в очень больших количествах. Вопрос: как создавать корпуса таких кораблей? На Земле целиком не соберёшь, сделать отдельные части на Земле и собрать в космосе тоже не получается. Приходится полностью собирать в космосе. Но в невесомости прокатный станок не соберёшь, литейной формы охуительных размеров тоже нет. Идея такова: выдувать полые металлические корпуса (примерно как на пикриле) из расплавленного металла прямо в космосе. Вместо гигантского стеклодува используем сверхмощные насосы и инертные газы. Станок получается циклопических размеров, но довольно простой. Получившийся полый корпус затем обрабатывается, заполняется другими деталями, получается космический корабль (со стороны кстати на астероид похожий). Вопрос: в чём сложности такой идеи, и не хуйня ли это с самого начала?
Аноним 12/07/20 Вск 12:24:09 588335491
>>588334
У тебя ложный премис. Во-первых, можно собрать прокатный станок на орбите. Но это и не нужно, ты можешь запулять стальные листы и сваривать их на орбите.
Но самое обидное и реалистичное, что пока что никому это нахуй не всралось, никто вообще никак не собирается практически летать даже на марс.
Аноним 12/07/20 Вск 12:49:30 588337492
>>588334
Литьё сильно ограничено термодинамикой - с отводом тепла и с электричеством в космосе серьёзные проблемы.

Для орбитальной сборки больших герметичных объемов наиболее реалистичной считается роботизированная сборка из небольших шестиугольных плиток с последующей их сваркой. Она позволяет получать любые формы. Были предложения потестить схему на практике, эксперимент поставить сравнительно несложно и вполне реализуемо.

Но есть трансформируемые конструкции (aka надувнушки), которые в принципе ставят под сомнение целесообразность орбитальной сборки гермоконструкций. Потому что они надёжнее (всё проверяется ещё на Земле, а сборную конструкцию-то придётся испытывать в космических условиях, что намного сложнее и вообще хз как делать) и дают набор герметичных объемов, т.е. как раз то что и нужно вместо одного гигантского. Просто задумайся, что космический корабль, как и морской, должен всё равно быть поделен на герметичные отсеки, чтобы при пробоине весь воздух не выдуло сразу. Так что проще собирать из готовых трансформируемых модулей.
Аноним 12/07/20 Вск 12:51:34 588338493
>>588337
>Потому что они надёжнее (всё проверяется ещё на Земле, а сборную конструкцию-то придётся испытывать в космических условиях
Ещё и потому, что трансформируемый модуль более-менее целен, а у плиток огромное количество швов, каждый из которых это точка отказа. Собственно как и у любой сборной конструкции.
Аноним 12/07/20 Вск 17:11:55 588364494
>>588334
Блядь, бигелоу дал вам надувные модули из брезентовой хуйни, которые можно хоть до размеров дирижабля надувать.
Аноним 12/07/20 Вск 19:20:50 588376495
>>588335
Листов надо дофига, запульнуть не получится. Корабль же огромный.
И да, разве в невесомости можно создать проектный станок?

>>588337
1. Разве тепло не излучается в космос
2. Надувные конструкции вряд-ли будут выдерживать условия космоса тысячелетиями.
Аноним 12/07/20 Вск 19:24:37 588378496
>>588376
>Листов надо дофига, запульнуть не получится.
Тебе один хуй надо дофига чего-то запульнуть, пушо в космосе детали просто так не летают. В долгосрочной перспективе (если делать ну просто дохуя всего надо), проще запульнуть завод и майнить руду на астероидах. Но это такое отдаленное будущее, что писец, пока из сотен нужных для этого технологий у нас есть примерно ноль.
Аноним 12/07/20 Вск 19:30:31 588379497
>>588378
Именно, я изначально планировал добывать руду на астероидах, плавить их (солнечным светом например) и выдувать из расплавленного металла корпуса.
Аноним 12/07/20 Вск 19:34:46 588380498
Аноним 13/07/20 Пнд 05:40:36 588427499
image.png 655Кб, 640x425
640x425
На мкс стоит винда? ЩИТО? Где вообще почитать что за ПО там стоит.
Аноним 13/07/20 Пнд 09:16:25 588432500
image.png 20Кб, 323x156
323x156
>>588427
Не, там в основном дебиан повсюду.
Аноним 13/07/20 Пнд 09:34:00 588434501
Безымянный.png 17Кб, 800x450
800x450
>>585265 (OP)
Все апполоны луну догоняли или шли наперерез?
Аноним 13/07/20 Пнд 09:36:02 588435502
image.png 31Кб, 350x184
350x184
image.png 272Кб, 850x567
850x567
image.png 422Кб, 650x433
650x433
image.png 999Кб, 1112x906
1112x906
Аноним 13/07/20 Пнд 12:50:56 588446503
>>588435
Переходная орбита у них в ту же сторону была, с запада на восток, так что скорее догоняли. «Наперерез» случался только в самом конце, чтобы при свободном возврате сделать восьмёрку и вернуться обратно к Земле, двигаясь в том же самом восточном направлении.

Так и на старте меньше топлива тратится (т.к. вращение Земли добавляет скорости), и при торможении требуется меньше теплозащиты (т.к. атмосфера движется в попутном направлении).
Аноним 13/07/20 Пнд 12:53:22 588447504
>>588446
Видимо, я не так вопрос понел. Разумеется на восток с добавлением скорости от земляшки, как иначе, они ж не из (((тех кто на запад ракеты запускает)))
Аноним 13/07/20 Пнд 13:22:02 588452505
>>588447
Им Тора на восток запускать не велит?
Аноним 13/07/20 Пнд 13:23:57 588453506
Я сейчас, наверное, совсем уж тупую хуйню спрошу, но у меня чёт ВРЁТИ и разрыв шаблона. Смотрел сегодня одного велопутешественника, он там едет по Перу через солончаки и кароч ориентироваться только по солнцу, ну он и спизданул что-то типа "Это же южное полушарие, тут солнце при движении с востока на запад отклоняется на север, а не как у нас на юг". Я что-то не так понял? Или он пизданул хуйню? Или это действительно так?
Таймкод- 10:30 https://www.youtube.com/watch?v=93RkIOrwyMk&t=1892s
Аноним 13/07/20 Пнд 13:25:57 588454507
>>588452
У них на востоке злые хачи, на них ракеты ронять чревато.
Аноним 13/07/20 Пнд 13:26:43 588455508
>>588453
Ну бля, Солнце над областью между тропиками, в районе экватора короч. Естественно, что от Перу это на севере.
Аноним 13/07/20 Пнд 13:26:51 588456509
>>588453
Все верно, у нормальных людей солнце на южной половине неба, а там на северной.
Аноним 13/07/20 Пнд 13:57:27 588460510
Как расширяется вселенная?
1. Если за пределами нет пространства, то ей ведь некуда расширяться. А так как расширяется пространство, то есть расширяется вселенная в саму себя? Или как это происходит в теории? То есть форма самой вселенной статична и сформирована?
2. Какой формы вселенная? Учитывая, что всё стремится к сфере - вселенная сферической формы?
Аноним 13/07/20 Пнд 14:05:40 588461511
>>588460
> Как расширяется вселенная?
С ускорением. Пока это всё.
Аноним 13/07/20 Пнд 16:18:47 588474512
>>588460
1. Само известное нам "пространство" находится внутри нашей вселенной. Раз расширяется — ну и пусть расширяется. Что находится за пределами — вселенной обсуждать смысла нет.
Аноним 13/07/20 Пнд 18:32:54 588504513
Объясните, почему громадная кинетическая энергия, которой обладает космический аппарат на орбите, не превращается в тепловую при спуске?
Если, например, столкнуть 2 стальных шара, они нагреются. Кинетическая энергия преобразуется в тепловую.
Почему же не нагреваются тела космонавтов, приборы и всё остальное внутри корабля?
Аноним 13/07/20 Пнд 18:34:11 588505514
>>588504
Еще как превращается, она вся уходит в нагрев воздуха и сжигание щита.
Потому, что они изолированы от плазмы, епт.
Аноним 13/07/20 Пнд 18:46:08 588508515
Почему никто не передает энергию двигателю ракеты через лазер с земли
Аноним 13/07/20 Пнд 18:51:20 588509516
>>588508
Хуйня получается, проще солярки с воздухом на борт взять и сжечь.
Аноним 13/07/20 Пнд 18:57:40 588510517
Аноним 13/07/20 Пнд 19:01:24 588511518
>>588510
Потому, что проще, заебал тупые вопросы задавать.
Аноним 13/07/20 Пнд 19:02:40 588512519
SS2168044.jpg 87Кб, 1199x434
1199x434
>>588504
>почему громадная кинетическая энергия, которой обладает космический аппарат на орбите, не превращается в тепловую при спуске?
Превращается. Спускаемый аппарат гонит ударную волну перед собой. В этой волне воздух почти моментально сжимается и от этого нагревается до состояния плазмы, разлагаясь на ионы. Типичная температура в наиболее горячих участках - 11-13 тыс градусов. Волна сама не касается аппарата, она бежит перед ним в нескольких сантиметрах. Но она всё равно передаёт большую часть своей энергии обратно аппарату, через тепловое излучение, т.к. раскалена до бешеной температуры.

>Почему же не нагреваются тела космонавтов, приборы и всё остальное внутри корабля?
Если речь о капсуле - у неё есть тепловой щит. Их бывает несколько разных типов.

Есть абляционный щит, он под действием высокой температуры испаряется с предсказуемой скоростью, постоянно испуская небольшое количество газа. Этот газ непрозрачен для инфракрасного (теплового) диапазона, и его непрерывный расходящийся в стороны поток образует тонкую подушку, отзеркаливающую излучение ударной волны наружу. Абляционные щиты используются для самых высокоскоростных аппаратов, входящих с межпланетных траекторий или прямо вниз, без планирования.

Аппараты с большой лобовой площадью относительно веса могут планировать, их можно не втыкать в атмосферу, а спускать под плавным углом, размазывая потерю энергии по более длинной траектории. Пиковый нагрев в таком случае меньше, и для отражения излучения достаточно плиток из специальной керамики, как у Шаттла, Бурана или X-37B.

Есть ещё раскрываемые замедлители (aka надувной щит, который на самом деле не всегда надувной, и вовсе не щит, а скорее парашют-замедлитель конической формы), у них огромная лобовая площадь и тормозят они очень хорошо, соответственно их можно спускать в планирующем режиме по совсем пологой траектории и размазывать потерю энергии по половине земного шара. Их можно делать из гибких материалов, например кевлара, который держит в районе 400 градусов. Таких пока было испытано полторы штуки, за пределами экспериментов и прототипов пока никто не применял - в основном не нравится низкая управляемость такого замедлителя, и низкая точность посадки.
Аноним 13/07/20 Пнд 19:07:25 588513520
>>588511
Ты мне зачем ответил, а? Чтобы повыпендриваться?
Аноним 13/07/20 Пнд 19:08:34 588514521
>>588508
Передавать хотели, только не лазером, а микроволновкой. Было даже несколько концептов. На бумаге это интересная идея, но:
- проблема с фокусировкой луча на больших дистанциях
- облачность и вообще любая влажность/вода в воздухе поглощают энергию луча
- теплообменник должен быть на пузе, если качнётся или луч промажет, то всё внутри сгорит к херам
- теплообменник не идеален, и греется всё равно весь аппарат
- по мере подъёма угол аппарата по отношению к лучу меняется, соответственно меняется и КПД
- потенциальные пиздарики мимоспутникам, по которым может скользнуть луч такой мощности, направленный в небо
- регенеративное охлаждение затруднено, т.к. большая эффективность означает большую температуру рабочего тела, но и меньшее количество рабочего тела, возимое с собой. Т.е. более эффективный аппарат всегда будет больше нуждаться в холоде, но будет возить с собой меньше холода.
Аноним 13/07/20 Пнд 19:32:39 588517522
>>588512
Это всё очень интересно, но не проясняет вопрос куда деваются 30 мегаджоулей каждого килограмма тушки огурца космонавта. Как она переходит в плазму-то, тем более если эта плазма даже не касается корпуса корабля?
Аноним 13/07/20 Пнд 19:35:13 588519523
>>588517
>куда деваются 30 мегаджоулей каждого килограмма тушки огурца
Ты горишь где-то минут пять.
30Мдж/300 секунд = 100 киловатт
Вполне резонная цифра для ебаной плазмы.
Аноним 13/07/20 Пнд 21:20:52 588574524
>>588517
Проясняет максимум детально.
>Спускаемый аппарат гонит ударную волну перед собой. В этой волне воздух почти моментально сжимается и от этого нагревается
Что именно тебе в этой фразе непонятно? Кинетическая энергия аппарата превращается в тепловую энергию волны сжатия.

>тем более если эта плазма даже не касается корпуса корабля
Это особенность ударной волны. Она возникает перед любым объектом, движущимся быстрее скорости звука в данной среде, и она тем дальше, чем больше разница их скоростей (число Маха). А аппарат при входе движется на ~22 махах.
Аноним 13/07/20 Пнд 21:25:25 588579525
>>588517
Деваются в нагрев ударной волны через сжатие. Можно входить под большим углом, плюхаясь в атмосферу отвесно - тогда торможение будет более коротким и горячим. Можно входить полого, едва касаясь плотных слоёв вначале - тогда торможение будет медленней и холодней.
Аноним 13/07/20 Пнд 21:26:04 588580526
>>588579
>Можно входить полого, едва касаясь плотных слоёв вначале - тогда торможение будет медленней и холодней.
Можно же случайно отскочить от атмосферы нахуй.
Аноним 13/07/20 Пнд 21:40:53 588582527
>>588574
>Спускаемый аппарат гонит ударную волну
Расширю формулировку вопроса:
Как кинетическая энергия с мизинца левой ноги космонавта Ивана Говнова, сидящего в спускаемом аппарате, переносится на эту самую волну? По воздуху? По эфиру? По щучьему велению?
Почему, блять, эта энергия не превращается в тепловую "на месте", то есть там где она уже есть. Откуда энергия знает что ей нужно воздух сжимать и эту ёбаную ударную волну создавать?
Аноним 13/07/20 Пнд 21:43:04 588583528
>>588582
Парень, ты... воопщи с ноший плонети? Космонавт давит на ложемент, ложемент на корпус, корпус на тепловой щит, который и сгорает.
Элементарно же. Хули космонавту воспламеняться, если реакция опоры действует весь путь?
Аноним 13/07/20 Пнд 21:54:07 588585529
>>588582
Короче не я один во всей этой хуйне сомневаюсь -
The Apollo Space Program – A gigantic conspiracy?
Wolfgang G. Gasser
http://www.pandualism.com/d/apollo.html
А вы дальше скачите жопами на ударных волнах своих.
Аноним 13/07/20 Пнд 21:56:15 588586530
>>588585
У тебя и самолеты не работают наверное, шизик?
Аноним # OP 13/07/20 Пнд 21:58:54 588588531
Halley1986byESO[...].jpg 68Кб, 800x1162
800x1162
Аноним 13/07/20 Пнд 22:37:00 588594532
>>588585
Иди с миром, прозревший анончес!

Пошел на хуй, поехавший.
Аноним 13/07/20 Пнд 23:35:29 588606533
Headofabarbaria[...].jpg 211Кб, 515x660
515x660
>>588582
>>588585
Блять, так бы сразу и сказал что ты шизоид, не осиливший 5 классов, не тратил бы на тебя время.
Аноним 13/07/20 Пнд 23:37:17 588607534
>>588580
Можно, поэтому такое снижение должно быть прецизионным. А в принципе можно и отскоками тормозить, это ещё плавнее, и ещё больше требования к управляемости и точности.
Настройки X
Ответить в тред X
15000
Макс объем: 40Mб, макс кол-во файлов: 4
Кликни/брось файл/ctrl-v
Стикеры X
Избранное / Топ тредов